You are on page 1of 96

宏景国际教育

LMCHK 香港职业医生
Hematology 4

INSTRUCTOR: Tony
l Bone marrow failure (e.g. aplastic anaemia, myelodysplastic
syndrome, drug-induced agranulocytosis)
l Hematological malignancies (e.g. acute leukaemia,
myeloproliferative disorders, lymphoproliferative disorders, multiple
myeloma)
l Blood transfusion
l Hematopoietic cell transplantation
l General oncology (including target and immunomodulatory
therapies)
l Paraneoplastic syndromes
l Palliative Care
• Characterised by pancytopaenia and a hypoplastic bone marrow
• Peak incidence of acquired = 30 years old Aplastic anemia
Features
• Assessment of bone marrow cellularity is best made on trephine biopsy(骨活
检), which often shows replacement of the normal cellular marrow by fatty
marrow.
• normochromic, normocytic anaemia
• leukopenia, with lymphocytes relatively spared
• Thrombocytopenia
• may be the presenting feature acute lymphoblastic or myeloid leukaemia
• a minority of patients later develop paroxysmal nocturnal haemoglobinuria or
myelodysplasia
• In patients with aplastic anemia, the bone marrow is markedly hypocellular.
Causes
• Idiopathic
• congenital: Fanconi anaemia, dyskeratosis congenita
• drugs: cytotoxics, chloramphenicol, sulphonamides, phenytoin, gold
• toxins: benzene
• infections: parvovirus, hepatitis
• radiation
This is a rare (~ 5 cases per million/year) stem cell disorder in which bone marrow
management
stops making cells, leading to pancytopenia. Presents with features of anaemia
Supportive
(↓Hb), infection (↓WCC ), or bleeding (↓platelets). Causes: Most cases are
• blood products
autoimmune, triggered by drugs, viruses (eg parvovirus, hepatitis), or irradiation.
• prevention and treatment of infection
May also be inherited, eg Fanconi anaemia . Tests: Bone marrow biopsy is
Anti-thymocyte globulin (ATG) and anti-lymphocyte globulin (ALG)
diagnostic. Treatment: Mainly supportive in asymptomatic patients. Transfuse
• prepared in animals (e.g. rabbits or horses) by injecting human lymphocytes
blood products as required and initiate neutropenic regimen if count <0.5*10⁹/L.
• is highly allergenic and may cause serum sickness (fever, rash, arthralgia),
The treatment of choice in young patients with severe disease is allogeneic
therefore steroid cover usually given
marrow transplantation from an HLA- matched sibling, which can be curative.
• immunosuppression using agents such as ciclosporin may also be given
Otherwise, immunosuppression with ciclosporin and antithymocyte globulin may
Stem cell transplantation
be effective, although it is not curative in most. There is no clear role for G-CSF.
 allogeneic transplants have a success rate of up to 80%
Q1. M/23 presented with severe aplastic anaemia. Which of
the following is the appropriate (best) treatment?
A. allogeneic bone marrow transplant if matched sibling donor
is available
B. anti-thymocyte globulin+ cyclosporin A
C. Corticosteroid
D. iron chelating therapy
Q3
E. rituximab

Q2 A 30+ yrs old lady was found to have severe pancytopenia Immunosuppressive therapy using antithymocyte globulin (ATG)
and was diagnosed to be aplastic anaemia. which of the and cyclosporine is associated with an overall response rate of
following is least effective? 60-80% and a 5-year survival rate of 75% in most reports, but
A. corticosteroid event-free survival rates are in the range of 35-50%.
B. anti-thymocyte antibody Immunosuppressive therapy using ATG plus cyclosporine is
C. cyclosporin being administered as first-line therapy for patients with severe
D. allogenic bone marrow transplant or very severe aplastic anemia (SAA or VSAA, respectively) who
E. plasmapheresis are older than 50 years (35-50 years in presence of
comorbidities) and as second-line therapy in younger patients
with SAA or VSAA if a human leukocyte antigen (HLA)–matched
sibling donor is not available. Immunosuppressive therapy is
also recommended in patients with nonsevere aplastic anemia
who are transfusion dependent.
Myelofibrosis myelofibrosis-most common symptom-lethargy
Tear drop poilkilocytes= myelofibrosis

Treatment: Bone marrow transplant is the only curative treatment

Blood film showing the typical 'tear-drop' poikilocytes


Q4 Myelofibrosis, will find in PBS: Pelger-Huët anomaly (PHA) is a benign, dominantly inherited
A. pencil cells defect of terminal neutrophil differentiation secondary to
B. tear drop cells mutations in the lamin B receptor (LBR) gene
C. Pseudo Pelger-Huet anomaly neutrophils with dumbbell-shaped, bilobed nuclei; a reduced
D. Giant platelets number of nuclear segments; and coarse clumping of the
nuclear chromatin.
pseudo–Pelger-Huët anomaly, which can be observed in
individuals with myeloid leukemia, myelodysplasia, and bi-
lineage acute lymphocytic leukemia, is important.
Pseudo-Pelger-Huet anomaly—neutrophils
with bilobed (“duet”) nuclei A . Typically seen
after chemotherapy.

Q5
Acquired neutropenia caused by drugs and chemicals, excluding cytotoxic chemotherapy(drug-induced agranulocytosis)
Numerous drugs have been associated with neutropenia. many drugs act by an immune-mediated mechanism. However, some drugs appear to
have direct toxic effects on marrow stem cells or neutrophil precursors in the mitotic compartment. Other drugs may have a combination of
immune and nonimmune mechanisms or may have unknown mechanisms of action. The highest risk categories are antithyroid medications,
macrolides, and procainamides.
Antimicrobials,Antipsychotics, Antihistamines,,Diuretics, Hypoglycemic,Antimalarial drugs, Heavy metals include gold, arsenic, and mercury.
Several drugs are particularly salient because of their high frequency of association with agranulocytosis:
l Phenothiazine
l Antithyroid drugs (thiouracil and propylthiouracil)
l Aminopyrine
l Chloramphenicol
l Sulfonamides
Neutropenic regimen (For when neutrophil count ≤ 0.5*10⁹/L.) Remove any offending drugs or agents in cases involving drug exposure: If the
identity of the causative agent is not known, stop administration of all drugs until the etiology is established . Close liaison with a microbiologist
and haematologist is vital. Abide by infection control procedures! Use a risk-assessment tool .
l Full barrier nursing in a side room if possible. Hand-washing is vital.
l Avoid IM injections (danger of an infected haematoma).
l Look for infection (mouth, axillae, perineum, IVI site). Take swabs.
l Check: FBC , platelets, INR , U&E , LFT , LDH , CRP . Take cultures (blood*3 — peripherally± Hickman line; urine, sputum, stool if
diarrhoea); CXR if clinically indicated.
l Wash perineum after defecation. Swab moist skin with chlorhexidine. Avoid unnecessary rectal examinations. Oral hygiene (eg hydrogen
peroxide mouth washes/ 2 h) and Candida prophylaxis are important .
l Check vital signs 4-hrly. High-calorie diet; avoid foods with high risk of microbial contamination. Vases containing cut flowers pose a
Pseudomonas risk.
Use of antibiotics in neutropenia. Treat any known infection promptly.
l If T>38 or T>37.5 on two occasions, >1h apart, or the patient is septic, start blind combination therapy according to local guidelines — eg
piperacillin–tazobactam—( + vancomycin, , if Gram+ ve organisms suspected or isolated, eg Hickman line sepsis). Continue until afebrile for
72h or 5d course, and until neutrophils >0.5*10⁹/L. If fever persists despite antibiotics, think of CMV , fungi (eg Candida; Aspergillus) and
central line infection.
Q6 The incidence of agranulocytosis of Methimazole ?
A. 1%
B. 5%
C. 0.5%
D. 15%
E. 0.1%

Q7 Which of the following drugs has the least incidence of


agranulocytosis?
A. Phenothiazine
B. Antithyroid drugs
C. Aminopyrine
D. Golden
E. Sulfonamides
Q8 Regard the agranulocytosis, which of the following is wrong?
A. Full barrier nursing in a side room if possible.
B. Avoid unnecessary rectal examinations
C. Im injection under sterile poses no extra-infection
D. macrolides pose higher risk of agranulocytosis than penicillin
E. Use of antibiotics in neutropenia. Treat any known infection promptly
l premalignant condition.
l primarily affects elderly people (>60).
Myelodysplastic syndrome (MDS)
l more common in males than in females
Pathophysiology
l clonal mutation predominates in the bone marrow, suppressing healthy stem cells.
l the main cause of cytopenias
Ø In the early stages of MDS→increased apoptosis (programmed cell death).
Ø As the disease progresses and converts into leukemia,→proliferation of leukemic cells overwhelms the healthy marrow.
Causes
l primary or idiopathic MDS (80%)
l genetic predisposition macrocytic anaemia, thrombocytopenia and
l hematopoietic stem cell injury caused by exposure to any of the following: neutropenia with a small number of circulating
Ø Cytotoxic chemotherapy
Ø Radiation blasts→suggests a diagnosis of myelodysplastic
Ø Viral infection syndrome
features
l 80% of patients present because of symptoms of anaemia (fatigue and malaise)
l Petechiae, ecchymoses, and nose and gum bleeding are common manifestations of a low platelet count.
l neutropenia may leads to fever and infections
l blood film:
Ø dimorphic picture (some red cells are hypochromic and microcytic, while others appear macrocytic)
Ø neutrophils are hypogranular and hyposegmented (Pseudo- Pelger-Huet cells).
Ø The peripheral blood count may show;
• single cytopenia (anemia, thrombocytopenia, or neutropenia) in the early phase or
• bicytopenia (2 deficient cell lines) or
• pancytopenia (3 deficient cell lines) in later stages.
Ø unexplained macrocytic anemia with no evidence of megaloblastic anemia
l Bone marrow aspirate stained with Perls' stain showed ring sideroblasts
Ø Ring sideroblasts contain an abnormally high concentration of iron, usually stored in perinuclear mitochondria.
Ø Perls' stain (which stains for iron) shows this iron deposition as a dark ring around the margin of the nucleus.
Ø Cytogenetic studies of the bone marrow cells:
• Chromosomal abnormalities are clonal and include 5q-, monosomy 7 (-7) or 7q-, trisomy 8 (+8),
ü Multiple combinations indicates a very poor prognosis.
ü A single abnormality, except those involving chromosome 7, indicates good prognosis.
Classification
l The (French-American-British (FAB) system classifies MDS into the following five subgroups :
Ø Refractory anemia (RA)
Ø RA with ringed sideroblasts (RARS)
u RA and RARS are characterized by≤5% myeloblasts in bone marrow.
u RARS is defined morphologically as having 15% erythroid cells with abnormal ringed sideroblasts,
u Both RA and RARS have a prolonged clinical course and a low prevalence of progression to acute leukemia.
ü progression to acute leukemia occurred in 5% of RARS cases, compared with 25% of RAEB cases
Ø RA with excess blasts (RAEB; 6-20% myeloblasts)
Ø RAEB in transition to AML (RAEB-T; 21-30% myeloblasts)
u acute myeloid leukemia (AML; >30%).
Ø Chronic myelomonocytic leukemia (CMML)
u manifests as
ü monocytosis of ≥ 1000/μL,
ü total white blood cell (WBC) count of < 13,000/μL, and trilineage dysplasia.
u CMML must be differentiated from classic chronic myelocytic leukemia, which is characterized by a negative Ph
chromosome.
l WHO classification 2008:
Ø Refractory anaemia with unilineage dysplasia- ie anaemia, neutropaenia or thrombocytopaenia (<5% blasts)
Ø Refractory anaemia with ring sideroblasts (<5% blasts; >15% sideroblasts)
Ø Refractory anaemia with multilineage dysplasia ( based on bone marrow dysplasia in 2 or more myeloid lineages)
Ø Refractory anaemia with excess blasts-1(5-9% blasts) and refractory anaemia with excess blasts -2 (10-19%)
u Blasts > 20% is now classified as acute myeloid leukaemia.
Ø Myelodysplasia unclassified
Ø Myelodysplasia with isolated 5qdel( cytogenetic abnormality with prognostic significance)
Prognosis
l Median survival is two years.
l Patients are more likely to have serious infections or life-threatening bleeds than blastic transformation.
l MDS who progress to acute leukemia have a poor prognosis than that of de novo acute myeloid leukemia (response to
chemotherapy is worse)
l International Prognostic Scoring System (IPPS)
Ø The revised I (IPSS-R) score is calculated on the basis of five variables:
1. Hemoglobin level
2. Absolute neutrophil count
3. Platelet count
4. Percentage of bone marrow blasts
5. Cytogenetic category
Management
l Supportive therapy,
Ø including transfusions of the cells that are deficient (ie, red blood cells [RBCs], platelets), and treatment of infections are
the main components of care.
Ø As the vast majority are elderly patients with other medical conditions, excessive intervention is unwarranted
Ø Granulocyte-colony stimulating factor (G-CSF) and recombinant erythropoietin (r-Epo) can improve blood counts.
u National Comprehensive Cancer Network (NCCN) guidelines recommend the use of erythropoiesis-stimulating
agents (ESAs) for treatment of symptomatic anemia in patients in the R-IPSS very low risk, low risk, or intermediate
risk category whose tumor lacks the 5q31 deletion and whose level of endogenous EPO is ≤500 mU/mL.
u In cases of the presence of ringed sideroblasts or an absence of response, the addition of granulocyte colony-
stimulating factor (G-CSF; filgrastim), 1–2 μg/kg 1–3 times per week should be considered.
Ø hypomethylating agent azacytidine, which has been shown to improve survival compared with either supportive or
aggressive therapy and is approved for use in MDS by (FDA).
Ø Aggressive cytotoxic chemotherapy is generally reserved for treatment of transformation to acute myelogenous
leukaemia (AML) in younger patients.
Q9 A 72-year-old man, DM for 8years on metformin, otherwise is well. fever, lethary, cough for 7days, chest X-ray, left low lobe
pneumonia. Lab test: Hb 76, wbc 2.8, plt 26, MCV 103, a few lymphocyte blasts was note in peripheral blood smear. LDH
108.what is the most likely diagnosis?
A. Pernicious anemia
B. Vitb12 deficiency anemia
C. aplastic anemian
D. Myelodyplastic syndrome
E. Folate deficiency anemia

Q10 About myelodyplatic syndrome, which of the followings is right


A. primarily affects both elderly people and children
B. hypersegmented neutrophils
C. Most cases have secondary causes
D. MDS who progress to acute leukemia have a poor prognosis than that of de novo acute myeloid leukemia
E. Chromosomal abnormalities involved chromosomal 7 indicates good prognosis
l Polycythaemia rubra vera (PRV) is a myeloproliferative disorder caused by clonal proliferation of a marrow stem cell leading to an increase in
red cell volume, often accompanied by overproduction of neutrophils and platelets.
l a mutation in JAK2 is present in approximately 95% of patients with PRV and this has resulted in significant changes to the diagnostic criteria.
l peak incidence in the sixth decade
Features
l Hyperviscosity
l pruritus, typically after a hot bath
l Splenomegaly
l haemorrhage (secondary to abnormal platelet function NOT NUMBER)
l plethoric appearance
l hypertension in a third of patients
l low ESR
l Low EPO levels
Ø the strongest pointer towards primary polycythemia
Ø myeloproliferative→ increased red blood cell production by the marrow→ turns off endogenous EPO production→ low EPO level.
l raised leukocyte alkaline phosphatase (ALP)
l Mild prolonged PT & PTT: this is related to the ratio of plasma and citrate. In the blue tubes that are used for coagulation tests the ratio is
normally 1 citrate to 9 of whole blood. If there is less plasma due to the polycythemia there will be excess citrate and this will prolong
coagulation tests such as the APTT and prothrombin time.
l Others: hyperuricaemia, peptic ulceration.
Investigations
Following history and examination, the British Committee for Standards in Haematology (BCSH) recommend the following tests are performed
l full blood count/film (raised haematocrit; neutrophils, basophils, platelets raised in half of patients
l JAK2 mutation
l serum ferritin
l renal and liver function tests
If the JAK2 mutation is negative and there is no obvious secondary causes the BCSH suggest the following tests:
l red cell mass
l arterial oxygen saturation
l abdominal ultrasound
l serum erythropoietin level
l bone marrow aspirate and trephine
l cytogenetic analysis
l erythroid burst-forming unit (BFU-E) culture
Diagnostic criteria
JAK2-positive PRV - diagnosis requires both criteria to be present
l High haematocrit (>0.52 in men, >0.48 in women) OR raised red cell mass (>25% above predicted)
l Mutation in JAK2
JAK2-negative PRV - diagnosis requires A1 + A2 + A3 + either another A or two B criteria
A1 Raised red cell mass (>25% above predicted) OR haematocrit >0.60 in men, >0.56 in women
A2 Absence of mutation in JAK2
A3 No cause of secondary erythrocytosis
A4 Palpable splenomegaly
A5 Presence of an acquired genetic abnormality (excluding BCR-ABL) in the haematopoietic cells
B1 Thrombocytosis (platelet count >450*10⁹/l)
B2 Neutrophil leucocytosis (neutrophil count > 10*10⁹/l in non-smokers; >12.5*10⁹/l in smokers)
B3 Radiological evidence of splenomegaly
B4 Endogenous erythroid colonies or low serum erythropoietin
Management
l Aspirin
l venesection - first line treatment
l hydroxyurea -slight increased risk of secondary leukaemia
l phosphorus-32 therapy
Prognosis
l thrombotic events are a significant cause of morbidity and mortality
l 5-15% of patients progress to myelofibrosis
Ø Pastest note→Transition from primary polycythaemia to myelofibrosis occurs in about 30%of patients, therefore, the
probability of developing myelofibrosis is higher and thus more likely than acute leukaemia
l 5-15% of patients progress to acute leukaemia (risk increased with chemotherapy treatment) particularly if patients have been
exposed to radioactive phosphorous treatment or busulfan therapy
Q11

Q.12: M/32, good past health, polycythemia vera. Mx?


A. venesection
B. venesection + warfarin
C. venesection + aspirin
D. venesection + aspirin + hydroxyurea
E. venesection + aspirin + anagrelide
Q13. A 40-year-old lady presented with thrombocytopenia was
found to have Hb 19g/dL. Which of the following is least
compatible with the diagnosis of polycythaemia rubra vera?
A. splenomegaly of 6cm
B. low erythropoietin level
C. high white cell count
D. severe thrombocytopenia
E. Philadelphia chromosome negative

Q14
Features
l asymptomatic (25-33% )
l tingling or burning in the hands and feet, headache, visual problems, weakness and dizziness.
Ø burning sensation in the hands is a characteristic symptom
Ø Erythromelalgia
n burning pain, warmth, and redness of the extremities
Ø The pain increases with exposure to heat and improves with cold
Ø These symptoms result from excessive numbers of platelets causing blockages in small or large blood vessels in different parts of the body.
l Other symptoms include sweating, low-grade fever, and pruritus.
l Splenomegaly (40-50%)
l Hepatomegaly (20% )
l both thrombosis and haemorrhage can be seen
Investigations
l Complete blood cell count (CBC)
Ø platelet count > 600 * 109/l
Ø Around 30% will also have a mildly raised RBC and / or WBC.
n A red blood cell (RBC) mass study helps to exclude polycythemia vera. The RBC mass is elevated in polycythemia vera, but is normal in essential
thrombocytosis.
l Genetic studies
Ø The majority of patients have mutations in one of three genes:
1. Janus kinase 2 (JAK2): 50-60% of patients.
2. calreticulin (CALR), found in 25%
3. myeloproliferative leukemia virus oncogene (MPL).:about 3-5% of cases. MPL codes for the thrombopoietin receptor protein, which promotes the growth and proliferation of
megakaryocytes. The mutations result in constitutive activation of the thrombopoietin receptor protein.
Ø Rare cases involve mutations in the thrombopoietin gene (THPO),: associated with autosomal dominant hereditary thrombocytosis
l Bone marrow examination
Ø bone marrow cellularity (found in 90% )
Ø Megakaryocytic hyperplasia is common
Ø Bone marrow reticulin is usually increased, but collagen fibrosis is uncommon
l Elevation of C-reactive protein (CRP), fibrinogen, and interleukin 6 levels suggests secondary thrombocytosis, because those are acute-phase reactants
l Vitamin B12 levels are increased in 25% of patients
l Uric acid levels are elevated in 25% of patients
Diagnosis
l British guidelines propose the following five criteria for diagnosis of essential thrombocytosis :
1. Sustained platelet count ≥450×109/L
2. Presence of an acquired pathogenetic mutation (eg, in the JAK2, CALR or MPL genes)
3. No other myeloid malignancy, especially polycythemia vera, primary myelofibrosis, chronic myeloid leukemia, or myelodysplastic syndrome
4. No reactive cause for thrombocytosis and normal iron stores
5. Bone marrow aspirate and trephine biopsy showing increased megakaryocyte numbers displaying a spectrum of morphology with predominant large megakaryocytes with
hyperlobated nuclei and abundant cytoplasm; reticulin is generally not increased (grades 0–2/4 or grade 0/3)
Ø Diagnosis requires the presence of criteria 1–3 or criterion 1 plus criteria 3–5.
adverse prognostic markers for essential thrombocythaemia (ET):
Age above 60; Symptomatology - particularly thrombosis and Platelet count above 1500.; Previous thrombosis; Obesity; Cardiovascular risk factors such as smoking, hypertension, and
hypercholesterolemia; Markers of hypercoagulability such as factor V Leiden and antiphospholipid antibodies ; JAK2 mutation
Management
l low risk observation only
l high-risk of thrombosis (eg, age >60, history of thrombosis, or platelet counts >1500 ).
Ø hydroxyurea (hydroxycarbamide) is widely used to reduce the platelet count: first-line treatment
Ø interferon-α is also used in younger patients
n Interferon alfa is a biologic response modifier.
n used as second line in older patient
n Interferon alfa is not known to be teratogenic and does not cross the placenta, perhaps making it safe for use during
pregnancy.
n Italian guidelines recommend interferon alfa as a first-line platelet-lowering therapy for patients younger than 40
years
Ø low-dose aspirin may be used to reduce the thrombotic risk
n low-dose aspirin may be useful in treating patients with symptoms of microvascular occlusion (eg, erythromelalgia).
n Patients with the JAK2 mutation or cardiovascular risk factors can be treated with daily low-dose aspirin
n Extreme thrombocytosis may promote the abnormal adsorption of large von Willebrand factor (VWF) multimers.
u These patients should be screened for the presence of acquired von Willebrand disease (VWD).
ü if ristocetin cofactorlevel(Functional von Willebrand Factor) is at least 30% in absence of other high risk
factors; Lowdose aspirin therapy (eg, ≤100 mg/day) is acceptable
ü if it is less than 30%, all aspirin should be avoided.
l Plateletpheresis
Ø If platelet is very high with symptoms of clotting or bleeding
Prognosis
l extremely good in ET with survival of over two decades expected.
l The risk of transforming to acute myeloid leukaemia is relatively low (<1%).
Q.15 Woman presents with shortness of breath on
walking upstairs. MCV 70, Hb5, WCC6 Plt 600,
reticulocyte 1%. What is your further management?
A. Check autoimmune markers
B. Check JAK2 V617F mutation
C. Thyroid function
D. Clotting profile
E. Per rectal exam, iron profile, Fecal occult blood test
Q17 Some person has 900x10⁹/L platelets, Hb a bit low
Q16 . Essential thrombocytosis treated with aspirin and
12g/dL, white cells normal. Also felt some pain in fingers
hydroxyurea. 8 days later, platelet normalised. MCV
What is the first thing you do?
increased from 89 to 102 fl. What is the cause?
A. Peripheral blood smear
A. Aspirin
B. Serum immunoglobulin
B. Hydroxyurea
C. JAK2 V617F mutation
C. Hypothyroidism
D. Serum erythropoietin
D. Progression to acute leukaemia
E. radioactive phosphorus
E. Progression to myelofibrosis
Haematological malignancies: genetics
Below is a brief summary of the common translocations associated with haematological malignancies
t(9;22)-Philadelphia chromosome
l present in> 95% of patients with CML
l this results in part of the Abelson proto-oncogene being moved to the BCR gene on chromosome 22
l the resulting BCR-ABL gene codes for a fusion protein which has tyrosine kinase activity in excess of normal
l poor prognostic indicator in ALL
t(15;17)
l seen in acute promyelocytic leukaemia (M3)
l fusion of PML and RAR-alpha genes
t(1:14)
l This translocation is associated with MALT (mucosa-associated lymphoid tissue) lymphoma and deregulates BCL10
t(8;14)
l seen in Burkitt's lymphoma
l MYC oncogene is translocated to an immunoglobulin gene
t(11;14)
l Mantle cell lymphoma
l deregulation of the cyclin D1 (BCL-1) gene
t(11; 18)
l This translocation is associated with MALT (mucosa-associated lymphoid tissue) lymphoma and deregulates MALT1
t(14;18)
l This translocation is associated with follicular lymphoma
l results in a chimeric heavy-chain Ig (chromosome 14) and BCL2 (chromosome 18) gene.
l This disease presents with painless “waxing and waning” lymphadenopathy in additional to constitutional symptoms
Haematological malignancies: infections
Viruses
l EBV: Hodgkin's and Burkitt's lymphoma, nasopharyngeal carcinoma
l HTLV-1: Adult T-cell leukaemia/lymphoma
l HIV-1: High-grade B-cell lymphoma
Bacteria
l Helicobacter pylori: gastric lymphoma (MALT)
Protozoa
l malaria: Burkitt's lymphoma
Q18

Q19 Which of the following has no relationships with infection?


A. gastric lymphoma
B. Burkitt's lymphoma
C. nasopharyngeal carcinoma
D. High-grade B-cell lymphoma
E. AML

Q20 About Philadelphia chromosome, which of the following is wrong?


A. the Abelson tumor supressor gene being moved to the BCR gene on chromosome 22
B. present in> 95% of patients with CML
C. BCR-ABL gene codes for a fusion protein which has tyrosine kinase activity
D. imatinib is now considered first-line treatment
E. Philadelphia chromosome bear a poor prognostic indicator in ALL
Acute myeloid leukaemia (AML )
l AML is the most common form of acute leukaemia in adults.
l It may occur as a primary disease or following a secondary transformation of a myeloproliferative disorder.
l Acute leukemia is defined as an accumulation of more than 20 percent of immature blasts at the bone marrow.
Ø Chronic myeloid leukaemia often ends in acute blastic transformation after a mean duration of approximately four years.
l classically associated with Down syndrome.
l Alkylating agents is a chemotherapy drug class that increases the risk of developing AML.
l characterized by cells with positive cytoplasmic staining for myeloperoxidase.
l The median age of onset of AML is 65 years.
Presentation
l Vague and non-specific (flu-like symptoms)
l Due to pancytopenia (Infection, anaemia , bleeding) ALL AML
l Splenomegaly may occur but typically mild and asymptomatic. Presence of auer rods in blood None Always
l LN swelling is rare.
l High total leucocyte count (TLC) leads to leucostasis and hyperviscositydrowsiness and retinal vein dilatation. Presnece of lymphoblasts in Always May or may
l Blood film reveals white cells predominantly myeloblasts and promyelocytes. blood not
Poor prognostic features
AMLCytogenetics Karyotype is of most prognostic value. Bone and joint pain More Less common
l >60 years common
l >20% blasts after first course of chemo
l cytogenetics: deletions of chromosome 5 or 7 Hepatosplenomegaly More Less common
Ø bone marrow cytogenetics are the most important aspect in determining prognosis in AML common
Good prognostic features
 Karyotype of bone marrow Organ infiltration More Quiet unusual
Ø patients with t(8:21) or chromosomes 16 inversion have a low risk of relapse common
Classification-French-American-British (FAB)
l MO – undifferentiated ALL cells characteristically stain positive for PAS
l M1 - without maturation
l M2 - with granulocytic maturation (Periodic acid-Schiff) and NSE (Non-specific Esterase).
Ø the most common (25% of adult AML)
Ø associated with a t(8;21) translocation. AML cells characteristically stain positive for Sudan
l M3-acute promyelocytic (APL)
Ø has the best prognosis of all the subtypes of AML.
Black and myeloperoxidase, but M4 and M5 cells
Ø Unlike the other AML subtypes, APL is treated with all-trans retinoic acid (ATRA). stain positive for NSE, while M6 cells stain positive for
Ø t(15;17)
l M4 - granulocytic and monocytic maturation PAS.
Ø associated with a t(16;16) translocation
l M5 – monocytic
Ø AML (monocytic) M5: high count of circulating blasts may lead to symptoms of cellular hyperviscosity (headache, confusion, fits, coma) and tissue deposits of leukaemia cells (gums
hypertrophy) with cells stain positive with Sudan Black and myeloperoxidase plus NES.
l M6 – erythroleukaemia; M7 – megakaryoblastic
Management: Combination chemotherapy including arabinosylcystosine after apheresis. Cytarabine and Anthracycline is considered the initial treatment of choice for patients with AML.
Bone marrow transplantation: The aim would be to choose a fully matched sibling who was also CMV-negative. In general, fully HLA matched, CMV matched, male donors are preferred over
Acute promyelocytic leukaemia (APML) the M3 subtype of AML.
l The importance of identifying APML lies in both the presentation (classically disseminated intravascular coagulation) and management
l APML is associated with the t(15;17) translocation
Ø causes fusion of the PML and RAR-alpha genes.
Ø In 95% of cases, retinoic acid receptor-alpha (RARA) gene on chromosome 17 is involved in a reciprocal translocation with the promyelocytic
leukaemia gene (PML) on chromosome 15.
Ø The mechanism underlying leukaemogenesis is aberrant fusion of 2 genes PML and RARA.
Features
l presents younger than other types of AML (average = 25 years old)
l DIC or thrombocytopenia often at presentation
l Auer rods (seen with myeloperoxidase stain)
Ø Auer rods are eosinophilic needle-like cytoplasmic inclusions found in blast cells
l good prognosis
management
l treatment of APML differs from that of all other AML forms
l the most appropriate initial treatment regimen: All trans retinoic acid (ATRA) a derivative of vitamin A., plus Anthracycline based chemotherapy
Retinoic acid syndrome (or differentiation syndrome)
Pathophysiology
l thought to be the result of the release of cytokines and subsequent lung infiltration by the neutrophils created by the maturation of myelocytes in APML.
l The presence of CD13 expression on leukemic cells can be a predictor of the future development of this syndrome.
Causes
l after treatment of APML with all-trans retinoic acid (ATRA) (present within a week of treatment)
l after treatment of APML with arsenic trioxide.
l usually occurs during induction therapy
Incidence
l 14-16% of patients.
Features
l dyspnea, pulmonary edema and effusions, A chest X-ray shows interstitial infiltrates.
l fevers,
l hypotension,
l Other complications include pericardial effusion, renal insufficiency, and hypertension.
treatment
l Corticosteroids
l the drug is temporarily stopped, then started again at 50-75% of the earlier dose. Alternatively, arsenic therapy can be tried.
prognosis
l Without prompt treatment with glucocorticoids, patients with this disorder have a mortality rate as high as 30% due to brain edema or hypoxemic respiratory
failure.
l Fortunately, most patients improve markedly within 12 hours and their symptoms resolved completely within 24 hours.
Q21 About auer rods, which of the following is wrong? Stains TRAP (tartrate-resistant acid phosphatase)⊕ (trapped
in a hairy situation). TRAP stain largely replaced with flow
A. clumps of granular material containing lysosomes,
cytometry. Associated with BRAF mutations.
peroxidase, lysosomal enzymes Hairy cell leukaemia
B. Associated with DIC l Hairy cell leukaemia is malignant proliferation disorder of B cells.
C. Auer rods may be seen in some types of AML l Rare, about 2% of leukemias.
D. eosinophilic needle-like cytoplasmic inclusions l more common in males (4:1)
E. seen with periodic acid–Schiff (PAS) stain l frequently occurs in men in their fifth decade.
Features
Q22 Match M0 M1 M2 M3 M4 M5 M6 M7 l Pancytopenia
A. the best prognosis of all the subtypes of AML l splenomegaly
B. gum hypertrophy l skin vasculitis in 1/3 patients
C. the most common adult AML l 'dry tap' despite bone marrow hypercellularity
D. cells stain positive for PAS. l tartrate resistant acid phosphotase (TRAP) stain positive
E. abnormal fusion protein: PML-RARA l characteristic hairy leukocyte on blood smear with a “fried egg”
appearance
l medium-sized lymphocytes with numerous spiky, peripheral,
Q23 All trans retinoic acid is indicator for which
cytoplasmic projections.
of the following?
Management
A. AML M1
l chemotherapy is first-line: cladribine ( adenosine deaminase
B. AML M3
inhibitor), pentostatin
C. ALL L1
Ø Cladribine is a purine analog→inhibit DNA polymerase and
D. ALL L3
cause DNA strand breaks.
E. Hairy cell leukaemia
Ø SE: myelosuppression, nephrotoxicity, and neurotoxicity.
l immunotherapy is second-line: rituximab, interferon-alpha
Chronic myeloid leukaemia (CML)
l The Philadelphia chromosome is present in more than 95% of patients with (CML).
l It is due to a translocation between the long arm of chromosome 9 and 22 - t(9:22)(q34; q11). This results in part of the ABL proto-
oncogene from chromosome 9 being fused with the BCR gene from chromosome 22. The resulting BCR-ABL gene codes for a
fusion protein which has tyrosine kinase activity in excess of normal
Presentation
l middle-age (40-50 years)
l anaemia,
l weight loss,
l splenomegaly may be marked abdominal discomfort
l spectrum of myeloid cells seen in peripheral blood
Ø The blood film shows both mature (neutrophils) and immature forms in various stages of differentiation (myelocytes and
metamyelocytes)
u In acute myelogenous leukemia (AML) one would expect only immature blasts.
l decreased leukocyte alkaline phosphatase
l may undergo blast transformation (AML in 80%, ALL in 20%)
Management
l Unlike (CLL), CML will progress to frank leukaemia quite rapidly, so treatment is needed.
l imatinib is now considered first-line treatment
Ø inhibitor of the tyrosine kinase associated with the BCR-ABL defect
Ø very high response rate in chronic phase CML
l If remission is not achieved with imatinib, then:
Ø in a patient under 60-65 years, an allogeneic transplant would be considered if there was a matched sibling donor;
Ø in a 50-year-old patient or younger a matched unrelated donor transplant would be considered too.
l If the patient had been in blast crisis phase, then AML-type chemotherapy as well as Glivec (imatinib) would be the choice.
l Hydroxyurea
l interferon-alpha
l allogenic bone marrow transplant
Allogenic bone marrow transplant
Complication
Cytomegalovirus pneumonia
l The microscopy shows owl’s eye inclusion bodies, characteristic of CMV, but diagnosis is usually made by PCR of blood/lavage fluid.
l It is the commonest life-threatening complication following allogenic bone marrow transplant,
l usually occurring within the first 4 months following surgery.
l the treatment of choice→Ganciclovir
l Onset is rapid and mortality in the context of BMT is around 80%, even with antiviral therapy (ganciclovir)
Q.24 30/M, good past health, presented with left upper quadrant
discomfort for 3 months. Pre-marital checkup, PE showed
splenomegaly
Investigation results (reference range given)
Hb: 12 Q25. 52/F, presented with abdominal distention. PE: mild
WBC: 105.5 (high) pallor, splenomegaly 4cm below the costal margin.
platelet: 640 (high) Blood test
Blood smear: neutrophil 55%, myelocytes 20%, basophils 10%, Hb 9.8 g/dL
5% blasts
MCV 96 fL
Most likely diagnosis
platelet 900 x 109/L
A. Acute myeloid leukaemia
B. Chronic myeloid leukaemia WBC 120 x 109/L
C. Chronic neutrophilic leukemia (myeloblast 1%, promyelocyte 1%, myelocyte 35%,
D. Essential thrombocythemia metamyelocyte 15%, granulocyte 45%, lymphocyte 1%,
E. Lymphoma with splenic involvement monocyte 1%, eosinophil 1%, basophil 2%)
Q26 25 yo/ F in check up A. Acute myeloid leukaemia
found raised WCC, rasised platelet B. Chronic myelomonocytic leukaemia
P/E found splenomegaly of 8 cm below umbilicus C. Chronic myeloid leukaemia
A. ALL D. Essential thrombocythaemia
B. APL E. Myelodysplastic syndrome
C. AML
D. CML
E. hogkin lymphoma
Q27
Q29

Q30 費城染色體,慢性粒細胞性白血病治療
A 骼氨酸激酶抑制劑-伊馬替尼
B 羥基脲
Q28 C 自體骨髓移植
D 異體骨髓移植
Q31 M/32. Splenomegaly 12cm. Blood results: Hb 12, WCC 133, Platelet 840. Neutrophil alkaline phosphatase
score 0. Diagnosis?
A. polycythemia rubra vera
B. chronic myeloid leukemia
C. essential thrombocythemia
D. chronic myelomonocyltic leukemia
E. myelofibrosis

Q32 25 yo/ F in check up


found raised WCC, rasised platelet
P/E found splenomegaly of 8 cm below
umbilicus
A. ALL
B. APL
C. AML
D. CML
E. hogkin lymphoma
Acute lymphoblastic leukaemia (ALL):
l (ALL) is a disease of children.
l Children with certain genetic and immunodeficiency syndromes are at
increased risk.
These include:
Ø Down syndrome,
Ø Neurofibromatosis type 1,
Ø Bloom syndrome(congenital telangiectatic erythema)
Ø ataxia telangiectasia.
l The most common presenting symptoms of ALL are nonspecific: fever,
infection, bleeding, bone pain, or lymphadenopathy.
l The 8:14 chromosomal translocation is associated with a particularly
poor prognosis, and is found in approximately 1% of adults with ALL.
The incidence of CNS involvement is very high at the point of diagnosis,
and median event free survival after chemotherapy is only two
months.
Treatment
l Before ALL treatment with chemotherapy, if blast cells count is very
high ˃100*10^9/l→the patient needs Leukapheresis to prevent sludge
in of capillary beds, this can be lifesaving.
l Central nervous system (CNS) therapy (intrathecal) is indicated in all
patients with ALL
l Lumber puncture (LP) should be delayed until chemotherapy has
begun
Q33 At increased risk of ALL, EXCEPT:
A. Down syndrome
B. Neurofibromatosis type1
C. Congenital telangiectatic erythema
D. Tuberous sclerosis
E. Ataxia telangiectasis

Q34 About ALL, Poor prognoostic factors except:


A. L3 type
B. T(8;14)
C. High initial wbc
D. L1 type
E. Male sex

Q35 2-year-old boy, ALL Type1, fatigue, fever for 3days, gum bleeding, bone pain. Hb:57g/L, PLT: 43*10⁹, WBC 110*10⁹; initial
Treatment should be?
A. Antibiotics
B. RBC Transfusion
C. PLT Transfusion
D. Leukapheresis
E. Analgesic
Chronic lymphocytic leukaemia (CLL)
l (CLL) is caused by a monoclonal proliferation of well-differentiated lymphocytes which are almost always B-cells (99%)
Prevalence
l CLL is the most common form of leukemia found in adults in Western countries.
l generally affects older populations (The median age at diagnosis is 72 years)
Features
l often none
l constitutional: anorexia, weight loss
l bleeding, infections
l lymphadenopathy more marked than CML
Complications
l hypogammaglobulinaemia leading to recurrent infections
Ø Infections are the most frequent complication causing death in patients with CLL.
Ø Although intravenous immunoglobulin prevents recurrent infections it does not prolong survival.
l warm autoimmune haemolytic anaemia in 10-15% of patients
l transformation to high-grade lymphoma (Richter's transformation)
Investigations
l blood film:
Ø smudge cells (also known as smear cells)
u smudge cells are the artifacts produced by the lymphocytes damaged during the slide preparation.
Ø ≥5000 monoclonal B lymphocytes/μl. The clonality of the circulating B lymphocytes needs to be confirmed by flow cytometry.
l Immuno-phenotyping will demonstrate the cells to be B-cells (CD19 positive). CD5 and CD23 are also characteristically positive in CLL.
Ø Peripheral blood flow cytometry is the most valuable test to confirm a diagnosis of CLL.
l Although a bone marrow biopsy is not required for diagnosis, it is recommended for the diagnostic evaluation of unclear
cytopaenias, or FISH or molecular genetics if peripheral blood cell lymphocytosis does not allow adequate
immunophenotyping
l An extended FISH analysis is recommended before the start of therapy because the detection of additional cytogenetic
abnormalities [del(11q) or trisomy 12] may have therapeutic consequences
Management CLL prognostic factors
l observation policy is usual during the early stages of the disease. Poor prognostic factors (median survival 3-5 years)
l Indications for treatment l male sex
l age >70years
Ø progressive marrow failure: the development or worsening of anaemia and/or l lymphocyte count > 50
thrombocytopenia l prolymphocytes comprising more than 10% of blood lymphocytes
Ø massive (>10 cm) or progressive lymphadenopathy l lymphocyte doubling time < 12 months
Ø massive (>6 cm) or progressive splenomegaly l raised LDH
l CD38 expression positive
Ø progressive lymphocytosis: > 50% increase over 2 months or lymphocyte l deletions of part of the short arm of chromosome 17 (-17p)
doubling time<6months Chromosomal changes
Ø systemic symptoms: l deletion of the long arm of chromosome 13 (-13q) is the most
u weight loss > 10% in previous 6 months, common abnormality, being seen in around 50% of patients. It is
u fever >38 C for > 2 weeks, associated with a good prognosis
l deletions of part of the short arm of chromosome 17 (-17p) are
u extreme fatigue, seen in around 5-10% of patients and are associated with a poor
u night sweats prognosis
u autoimmune cytopaenias e.g. ITP Differential diagnosis
l Drugs(氟达拉滨 阿糖腺苷的氟化核苷酸类似物,其代谢产物可以通过抑制核苷酸还原酶、DNA聚合酶α、δ和ε,DNA引物酶和DNA连接酶从而抑 l mantle cell lymphoma (MCL)
制DNA的合成。此外,还可以部分抑制RNA聚合酶Ⅱ从而减少蛋白的合成),
Ø These tumour cells express B-cell surface antigens and also
expresses CD5, but usually not CD23.
Ø Fludarabine, cyclophosphamide and rituximab (FCR) has now emerged as the Ø For cases that express CD23, staining for cyclin D1 or SOX11
initial treatment of choice for the majority of patients and fluorescence in situ hybridisation (FISH) for detecting a
Ø monitoring by regular blood counts translocation (11;14) are useful for establishing the
Ø What antimicrobial prophylaxis should he receive before starting diagnosis of MCL.
chemotherapy with fludarabine? l small lymphocytic lymphoma (SLL)
Ø In the World Health Organization classification, small
u Co-trimoxazole lymphocytic lymphoma (SLL) and CLL are considered to be a
ü Fludarabine is a purine analogue that is phosphorylated single entity.
intracellularly. Ø The diagnosis of SLL requires the presence of
ü All of the purine analogues cause myelosuppression, but there is a lymphadenopathy and/or splenomegaly with a number of B
significantly higher risk of patients developing Pneumocystis lymphocytes in the peripheral blood not exceeding 5×10⁹/l.
Ø SLL cells show the same immunophenotype as CLL.
jirovecii pneumonia while on treatment. Ø The diagnosis of SLL should be confirmed by
ü Use of prophylactic co-trimoxazole (Septrin) has dramatically histopathological evaluation of a lymph node biopsy,
reduced the frequency of this severe opportunistic infection in whenever possible.
these patients. l monoclonal B-lymphocytosis’ (MBL)
Ø In absence of lymphadenopathy, organomegaly, cytopaenia
ü Co-trimoxazole should be continued after chemotherapy until the and clinical symptoms, the presence of fewer than 5000
CD4 counts exceeds 200 cells/mm3 (0.2×10⁹/L). monoclonal B lymphocytes/µl defines ‘monoclonal B-
l Regular infusions of immunoglobulin to prevent infections lymphocytosis’ (MBL)
l Recurrent infections are recognised in CLL due to hypogammaglobulinaemia and Ø can be detected in 5% of subjects with normal blood count.
immune paresis; but are not an indication for disease control. Ø Progression to CLL occurs in 1%–2% of MBL cases per year
Q36 Someone was admitted with bilateral cervical
lymphadenopathy, splenomegaly to 3cm below costal margin.
Leucocytosis (WBC up to 180) with anaemia and
thrombocytopenia. Blood smear showed smear cell. Checked,
CD7 and CD 20 present. But some other CD and sth else is not
present, what is the most likely diagnosis?
A. CLL
B. CML
C. ALL
D. Follicular lymphoma in leukaemic phase
E. Mantle cell lymphoma in leukaemic phase

Q37 Old man blood picture shows normal Hb, normal platelet Q38. A 70-year-old woman has a blood result showing
counts but high lymphocyte counts. Peripheral blood smear lymphocytosis of 10x10^9/L. Which of the following is an
shows abundant small mature lymphocytes. Most likely cause? appropriate diagnosis?
A. Tuberculosis A. chronic lymphocytic leukaemia
B. Viral infection B. CMV infection
C. CLL C. infectious mononucleosis
D. CML D. Pertussis
E. Mycoplasma infection E. tuberculosis
副蛋白血症

In the interpretation of serum protein electrophoresis, most attention focuses on


the gamma region(gamma-globulin zone), which is composed predominantly of
antibodies of the IgG type.
Multiple myeloma:classic symptoms : bone pain, pathological fracture, anaemia and hypercalcaemia (leading to
thirst).Multiple myeloma causes a low anion gap.
Definition
l Multiple myeloma is a neoplasm of the bone marrow plasma cells.
Epidemiology
l The peak incidence is patients aged 60-70 years.
l Multiple myeloma is the most common primary tumor of the bone in patients older than 50 years.
l equal sex ratio
l more common in Afro-Caribbean ethnic groups than in Caucasians
Monoclonal products produced
l IgG (50-60%)
l IgA (20-30%)
l light chain disease (20%)
Association
l Type2/Proximal renal tubular acidosis is a type of renal tubular acidosis associated with multiple myeloma.
Pathophysiology
l Neoplastic proliferation of plasma cells
l Bone marrow infiltrationsuppression of hematopoiesis→leukopenia,thrombocytopenia, anemia
l Cell proliferation→osteolysis→hypercalcemia
l Overproduction of monoclonal immunoglobulin and/or light chains
Ø Non-functioning antibodies→functional antibody deficiency
Ø Serum viscosity→hyperviscosity syndrome
Clinical features
l bone disease:
Ø due to neoplastic plasma cells activating RANKL receptors on osteoclasts.
Ø bone pain, (Bones commonly affected are the flat bones of the spine, and as such lower back pain is one of the most common presenting
features) Which acid-base disorders may be found
Ø osteoporosis + pathological fractures (typically vertebral), osteolytic lesions in an IgG multiple myeloma?
Ø weakness and paresthesias in the lower extremities due to vertebral compression fractures Low anion-gap metabolic acidosis.
l anaemia IgG tends to be cationic, whereas IgA
Ø fatigue and malaise tends to be anionic. As a consequence,
patients with IgG myeloma will tend to
Ø The most common presenting manifestations of multiple myeloma are those related to anemia. have a lower than normal serum anion
l Infection gap.
l hypercalcaemia→nausea, fatigue, confusion, polyuria, constipation
l Hyperphosphataemia: due to reduced renal excretion which may be directly due to renal impairment or interference with excessive protein
load.
l Foamy urine: caused by Bence Jones proteinuria
l renal failure
Ø the most common cause is from light chain deposition.
Ø Usually, the renal damage in MM is tubular. Occasionally there may be glomerular damage with consequent albumin loss.
l amyloidosis e.g. Macroglossia, carpal tunnel syndrome; neuropathy; hyperviscosity
Ø carpal tunnel syndrome - the most common peripheral neuropathy associated with multiple myeloma
l Multiple myeloma may present with roleaux formation on blood film and raised total protein (globulin component).
Ø The globulin level is markedly raised (albumin + globulin = total protein), suggesting the presence of a paraprotein.
Ø (globulin level = total protein ). A normal level should be below 36 g/L.
l Hypercalcaemia in myeloma
Ø primary factor: due primarily to increased osteoclastic bone resorption caused by local cytokines (e.g. IL-1, tumour necrosis factor)
released by the myeloma cells
Ø much less common contributing factors: impaired renal function, increased renal tubular calcium reabsorption and elevated PTH-rP
levels
Diagnostic criteria for multiple myeloma requires one major and one minor criteria or three minor criteria in an individual who has signs or
symptoms of multiple myeloma.
l Major criteria
Ø Plasmacytoma (as demonstrated on evaluation of biopsy specimen)
Ø 30% plasma cells in a bone marrow sample
Ø Elevated levels of M protein in the blood or urine
u monoclonal proteins: in the serum→(usually IgG or IgA); in the urine (Bence Jones proteins) : there is Negative dipstick for protein
and positive in biochemistry, because Bence jones proteins are not detected by dipstick
l Minor criteria Smoldering multiple myeloma→multiple myeloma (M-protein >3g/dL or
Ø 10% to 30% plasma cells in a bone marrow sample. >10% plasma cells in bone marrow) + no end organ damage.
criteria for end-organ damage, which are:
Ø Minor elevations in the level of M protein in the blood or urine. • Serum calcium >11.5 mg/dL
Ø Osteolytic lesions (as demonstrated on imaging studies). • Serum creatinine >2 mg/dL or estimated creatinine clearance <40 ml/min
Ø Low levels of antibodies (not produced by the cancer cells) in the blood. • Anemia with hemoglobin <10 g/dL
Investigations: (NICE 2016) • Bone lesions: osteolytic, pathological fracture; osteopenia
1. to confirm the presence of a paraprotein indicating possible myeloma or (MGUS): Treatment: Observe and monitor
Ø serum protein electrophoresis and serum-free light-chain assay→(best initial test)→serum protein electrophoresis
Ø If serum protein electrophoresis is abnormal→use serum immunofixation
Ø Do not use serum protein electrophoresis, serum immunofixation, serum-free light-chain assay or urine electrophoresis (urine
Bence–Jones protein assessment) alone to exclude a diagnosis of myeloma.
2. to confirm a diagnosis of myeloma: Non-secretory myeloma: Bone marrow clonal plasma cells
Ø bone marrow aspirate and trephine biopsy =10%, Myeloma-related end-organ damage, No M protein in
blood or urine
Ø the bone marrow aspirate would confirm the diagnosis irrefutably.
u morphology to determine plasma cell percentage
ü plasma cells greater than 4% and usually greater than 30%.
u flow cytometry to determine plasma cell phenotype
u bone marrow aspirate→dark red jelly-like material in the syringe (Plasma cells )
3. in a patient presenting with spinal cord compression:
Ø the most appropriate initial investigation is→Urgent MRI of her spine
u This should be done before investigation that used to confirm myeloma.
Ø skeletal survey→bone lesions
Treatment : general view
l The best initial treatment of multiple myeloma is chemotherapy induction.
l autologous bone marrow transplant in addition to chemotherapy has better results than chemotherapy alone.
l Asymptomatic patients:→watch and wait, unless patients have:
Ø ≥ 60% clonal cells,
Ø excessive free light chains or
Ø ≥1 bone lesion
l Symptomatic patients
Ø HCT eligible : induction therapy followed by autologous HCT
Ø HCT ineligible: chemotherapy alone (e.g., dexamethasone and lenalidomide)
l Supportive therapy
Ø Osteolysis and bone pain
u Bisphosphonates
u Radiation therapy of osteolytic regions
Ø Pancytopenia with anemia and increased risk of infection
u Blood transfusions
u Granulocyte-colony stimulating factor (G-CSF) and erythropoietin (EPO)
Myeloma: prognosis
l B2-microglobulin is a useful marker of prognosis - raised levels imply poor prognosis.
Ø Beta-2-microglobulin has been shown to be predictive of risk of progression of disease in myeloma, myelodysplastic
syndrome, and chronic myeloid leukaemia.
Ø In myeloma it is an accurate estimate of total disease load, with guidelines suggesting that a beta-2-microglobulin level of
>3.5 mg/L is strongly associated with increased mortality and morbidity.
l Low levels of albumin are also associated with a poor prognosis
l Increased lactate dehydrogenase levels more than double the normal is considered a bad prognostic sign in multiple myeloma.
Treatment (NICE 2016)
l previously untreated multiple myeloma ( newly diagnosed)
Ø Patients who are eligible for high-dose chemotherapy with stem cell transplantation
u bortezomib + dexamethasone,
u or bortezomib + dexamethasone + thalidomide
Ø if high-dose chemotherapy with stem cell transplantation is considered inappropriate
u thalidomide + alkylating agent + corticosteroid
l People who are at first relapse having received one prior therapy and who have undergone, or are unsuitable for, bone marrow transplantation:
Ø bortezomib (a proteasome inhibitor) monotherapy
l People who have received two or more prior therapies:
Ø lenalidomide + dexamethasone
u lenalidomide→immunomodulatory derivatives (structural derivatives of thalidomide)
l People with untreated, newly diagnosed, myeloma-induced acute renal disease:
Ø bortezomib + dexamethasone
Ø If a bortezomib is unsuitable→thalidomide + dexamethasone
Ø Do not perform plasma exchange for myeloma-induced acute renal disease.
l Preventing bone disease, managing non‑ ‑ spinal and spinal bone disease
Ø bisphosphonates should be given routinely, even in the absence of hypercalcaemia.
Ø Bisphosphonates reduce bony disease in myeloma, lowering the frequency of pathological fractures, modulate the disease and have some antitumour
activity.
u zoledronic acid or
u disodium pamidronate, if zoledronic acid is contraindicated or not tolerated or
u sodium clodronate, if zoledronic acid and disodium pamidronate are contraindicated, not tolerated or not suitable
Ø surgical stabilisation followed by radiotherapy for non-spinal bones that have fractured or are at high risk of fractures.
Ø Consider radiotherapy for people who need additional pain relief
l Managing peripheral neuropathy 硼替佐米是哺乳动物细胞中26S蛋白酶体糜蛋白酶样活性的可逆抑制剂。26S蛋白酶体是一种大的蛋白质复
Ø If patient on bortezomib 合体,可降解被泛素化的蛋白质。泛素蛋白酶体通道在调节特异蛋白在细胞内的浓度中起到重要作用,以
u switch to subcutaneous injections and/or 维持细胞内环境的稳定。蛋白水解会影响细胞内多级信号串联,这种对正常细胞内环境的破坏会导致细胞
u reduce to weekly doses and/or 的死亡。而对26S蛋白酶体的抑制可防止特异蛋白的水解。体外试验证明硼替佐米对多种类型的癌细胞具有
u reduce the dose. 细胞毒性。临床前肿瘤模型体内试验证明硼替佐米能够延迟包括多发性骨髓瘤在内的肿瘤生长。
Ø if patient on other than bortezomib
u Temporarily stop neuropathy-inducing myeloma treatments if people develop either of the following:
ü grade 2 neuropathy with pain
ü grade 3 or 4 neuropathy
l Managing fatigue
Ø Erythropoietin analogues (adjusted to maintain a steady state of haemoglobin at 110–120 g/litre) to improve fatigue in people with myeloma who have
symptomatic anaemia.
l Cord compression secondary to bony involvement of multiple myeloma:
Ø I.V Steroids should be commenced immediately
Ø However, the treatment of choice is local radiotherapy. NICE suggest localised radiotherapy should be the first point of call for urgent treatment.
Ø Vertebroplasty is typically considered in patients of whom have evidence of metastatic changes in the spine, but show no signs of spinal cord compression.
Ø Surgical decompression: is also considered if imaging suggests any form of spinal instability or structural defects, but often after steroids and radiotherapy
has been administered.
Ø Other treatment options include analgesia, with non-steroidal anti-inflammatory drugs of particular use.
Q.39 A question about a 8x woman with chest pain and fever.
Chest radiograph showed multiple rib fracture. Investigations
shows, pancytopenia and raised creatinine. Peripheral blood
smear showed rouleaux formation. What is the best next step?
A. Bone scan
B. CD4/8 lymphocyte subset counts
C. whole body PET
D. Serum B2-microglobulin
E. Urine and serum protein electrophoresis Q41 F/86, 2 weeks of fever, rib pain and confusion. CXR showed
pneumonia and multiple fractures of ribs. HyperCa, renal
impairment and Hb = 8d/L
Q40 Some old lady was admitted. Have multiple rib fracture, A. Bone scan
hypercalcemia, anaemia, and renal failure. What is your B. Renal biopsy
investigation of diagnostic value? C. Echo
A. Bone marrow biopsy D. Bone density study
B. Bone scan E. Bone marrow exam
C. Renal biopsy
D. PET CT
E. Echocardiogram
Q42. 78/F present with fever, dehydration, back pain and
confusion. CXR shows left lower lobe bronchopneumonia.
Investigtaion results
Hb 7.8 g/dL
WBC 10 x 109/L
Platelet 110 x 109/L
Calcium 3.1 mmol/L [2.24-2.63]
Blood film shows rouleaux formation. Q44 78/M with past history of DM, C/O dizziness. Blood results
What is the next investigation? as follows, Hb 6.5, WBC normal, Cr 180, Alb/Globulin: 32/72
A. Bone density scan The likely diagnosis:
B. Bone scan A DM nephropathy
C. Bronchoscopy B Variceal bleeding from underlying cirrhosis
D. Lymphocyte B-cell/T-cell subset count C AML
E. Serum and urine protein electrophoresis D Multiple myeloma

Q43 Man with chest pain was found to have multiple rib
fractures. Blood smear shows rouleaux formation. Most
appropriate next step to do:
A. Bone marrow examination
B. Bone scan
C. Skeletal survey
D. Renal biopsy
E. Chest CT
Q45 M/70, presented with bone pain and diagnosed multiple
myeloma with multiple lytic bone lesion:
A. he will benefit from bisphosphonate treatment
B. he will benefit from allo-HSCT
C. he has cryoglobulinemia
D. he has autoimmune hemolytic anemia
E. he has raised IgE

Q46 A 55/M 6cm spleen found incidentally on exam, which is


least likely?
A. multiple myeloma
B. CML
C. Myelofibrosis
D. NHL
E. Hb H disease
Monoclonal gammopathy of undetermined significance (MGUS )
l MGUS also known as benign paraproteinaemia and monoclonal gammopathy) is a common condition that causes a
paraproteinaemia and is often mistaken for myeloma.
l can be seen in >5% of people over 70 years of age.
Risk of transmission to malignancy:
l Around 10% of patients eventually develop myeloma at 5 years, with 50% at 15 years
l 1 percent per year develop multiple myeloma.
Features
l usually asymptomatic
l no bone pain or increased risk of infections
l around 10-30% of patients have a demyelinating neuropathy
Differentiating features from myeloma
l normal immune function
l normal beta-2 microglobulin levels
l lower level of paraproteinaemia than myeloma (e.g. < 30g/l IgG, or < 20g/l IgA)
l stable level of paraproteinaemia
l no clinical features of myeloma (e.g. lytic lesions on x-rays or renal disease)
Treatment
l Observation
l if there is neuropathy
Ø MGUS patients are associated with osteoporosis and osteopenia. They may benefit from treatment with bisphosphonates
u pyrophosphate analogue
u act by binding to hydroxyapatite in bone which leads to low osteoclastic activity.
Q47 A 72-year-old man C/O persistent tiredness over the past
3 months. No other abnormality. Investigations reveals Albumin:
38 g/l, IgG paraprotein band: 14 g/l, Bone marrow: 7% plasma
cells. Which of the following is the most appropriate
intervention? MGUS is defined by paraprotein (<30 g/l), bone
A. Observation marrow plasma cells <10% and the absence of
B. bortezomib + dexamethasone myeloma-related organ or tissue damage
A. thalidomide + alkylating agent + corticosteroid (predominantly renal, skeletal or bone marrow
C. Lenalidomide impairment). Annual overall progression to
D. Bortezomib myeloma is 1% and, as such, no intervention is
required.
l Also known as Lymphoplasmacytoid lymphoma
l most common in older white men
Pathophysiology
l monoclonal IgM production by a malignant lymphoplasmacytic clone that can cause damage to multiple organs.
l The tumor cells in Waldenstrom macroglobulinemia are positive to CD20 markers.
Features
l monoclonal IgM paraproteinaemia
l systemic upset: weight loss, lethargy
l hyperviscosity syndrome e.g.:
Ø visual disturbance,
Ø neurological symptoms such as headache, dizziness, and vertigo raynaud phenomenon
Ø Bleeding is a possible complication as viscous serum causes defective platelet aggregation.
l Hepatosplenomegaly
l Lymphadenopathy
l cryoglobulinaemia e.g. Raynaud's
Investigations
l protein electrophoresis→elevated IgM
l Bone marrow biopsy (the gold standard for the diagnosis)
Ø Shows→abnormal plasma cells with Dutcher bodies(intranuclear inclusions of IgM deposits) Dutcher bodies: PAS染色阳性包涵体
Differential diagnosis
l multiple myeloma
Ø usually presents with IgG or IgA secretion and lytic bone lesions.
Treatment
l Treatment only indicated in symptomatic patients
Ø asymptomatic→Follow-up
l Causative: CD20 antibodies (e.g., rituximab)
l Hyperviscosity syndrome: plasmapheresis
Q48 83-year-old white men with history of hypertension and DM, lethargy, weight
lose for 3months, headache, dizziness, , vertigo,and visual disturbance for 5 days.
Painful fingers in cold water. Cervical Lymphadenopathy, hepatosplenmegaly. Bone
marrow biopsy:abnormal plasma cells with Dutcher bodies. serum protein
electrophoresis, abnomal the gamma region(gamma-globulin zone) will be?
A. IgG
B. IgA
C. IgD
D. IgM
E. IgE

Q49 gamma-globulin excess, bone marrow find plasma cell infiltration, lymphoplasmacytic
cell infiltration. What is the diagnosis?
A. Multiple myeloma
B. Waldenstrom's macroglobulinaemia
C. Monoclonal gammopathy of undetermined significance
D. Heavy chain disease
E. Chronic lymphocytic leukemia
Hodgkin's lymphoma (HL):
l Hodgkin's lymphoma is a malignant proliferation of lymphocytes characterised by the presence of the Reed-Sternberg cell.
l hematological malignancy arising from mature B cells.
l Lymphadenopathy, typically painless and most commonly involving the cervical and/or supraclavicular nodal chain, is the most common
presenting symptom of HL.
Epidemiology
l It has a bimodal age distributions being most common in the third and seventh decades
Risk factors
l history of EBV infection
l family history of Hodgkin's lymphoma
l young adults from higher socio-economic class
l Immunodeficiency: e.g., organ or cell transplantation, immunosuppressants, HIV infection , chemotherapy
l Autoimmune diseases (e.g., rheumatoid arthritis, sarcoidosis)
Features
l Painless lymphadenopathy
Ø Most common is cervical lymph nodes (in ∼ 60–70% of patients)
l Mediastinal mass→ chest pain, dry cough, and shortness of breath
l Splenomegaly or hepatomegaly may occur if the spleen or liver are involved.
l B symptoms
Ø Night sweats,
Ø weight loss > 10% in the past 6 months,
Ø fever > 38°C (100.4°F)
l Can occur in a variety of diseases, such as non-Hodgkin lymphoma, other malignancies, tuberculosis, and various inflammatory diseases
l Pel-Ebstein fever
Ø Intermittent fever with periods of high temperature for 1–2 weeks, followed by afebrile periods for 1–2 weeks. Relatively rare but very
specific for HL.
l Alcohol-induced pain
l Pruritus (focal or generalized)
Histological classification

Poor prognosis
l weight loss > 10% in last 6 months
l fever > 38 C
l night sweats
l Other factors associated with a poor prognosis identified
in a 1998 NEJM paper included:
Ann-Arbor staging of Hodgkin's lymphoma Ø age > 45 years
l I: single lymph node Ø stage IV disease
l II: 2 or more lymph nodes/regions on same side of diaphragm Ø haemoglobin < 10.5 g/dl
l III: nodes on both sides of diaphragm Ø lymphocyte count < 600/l or < 8%
Ø Spleen is regarded as a Lymph Node region, So lymphoma with splenomegaly: Stage III
Ø Male
l IV: spread beyond lymph nodes
Each stage may be subdivided into A or B Ø albumin < 40 g/l
l A = no systemic symptoms other than pruritus Ø white blood count > 15,000/l
l B = weight loss > 10% in last 6 months, fever > 38c, night sweats (poor prognosis) Ø A mass of >10 cm in size
Fatigue, pruritus, EBV infection although they are common, BUT they have no prognostic significance.
Diagnosis
l Lymph node biopsy would be more likely to be
positive, RSC is evident on microscopy.
l Bone marrow
Ø Hodgkin results in patchy bone marrow
infiltration, an isolated bone marrow biopsy
may yield non-specific results.
Ø Bone marrow biopsy is more useful for staging
of advanced disease
Management:
l Early stage (IA or IIA): Radiotherapy and
chemotherapy.
l Later stage (III, IVA or IVB): Chemotherapy alone.
l Large mass in chest regardless of stage: Radiotherapy
and chemotherapy.
l Chemotherapy includes ABVD: Adriamycin (also
known as Doxorubicin), Bleomycin, Vincristine,
Doxorubicin, cyclophosphamide, prednisolone,
Rituximab & others
l Relapsed Hodgkin lymphoma→salvage chemotherapy
followed by BEAM conditioned autologous stem cell
transplantation as the established gold standard.
Non-Hodgkin's lymphoma (NHL) (NICE guideline 2016)
l include any kind of lymphoma except Hodgkin’s lymphomas.
l Most of NHL are of B cell phenotype, although T cell tumours are increasingly being recognized.
l subtypes of non-Hodgkin's lymphoma (NHL):
Ø diffuse large B-cell lymphoma
Ø Burkitt lymphoma.
Diagnosis
l Type of biopsy:
Ø first line→ excision biopsy
Ø if not surgically feasible→ needle core biopsy procedure
l in patient with histologically high-grade B-cell lymphoma:
Ø use FISH (fluorescence in situ hybridisation) to identify a MYC rearrangement
Ø If a MYC rearrangement is found,→use FISH to identify the immunoglobulin partner and the
presence of BCL2 and BCL6 rearrangements.
l Indications of using FDG-PET-CT imaging (fluorodeoxyglucose-positron emission tomography-CT)
Ø Staging
Ø to assess response at completion of planned treatment for:
udiffuse large B‑ cell lymphoma
uBurkitt lymphoma.
Ø to assess response to treatment before autologous stem cell transplantation for high-grade (NHL).
Non‑ Hodgkin lymphoma: Neoplasms of mature B cells

Non‑ Hodgkin lymphoma:Neoplasms of mature T cells


Adult T-cell lymphoma: Caused by HTLV Mycosis fungoides/ Sézary syndrome
(associated with IV drug abuse) Mycosis fungoides: skin patches and plaques (cutaneous T-cell lymphoma),
Adults present with cutaneous lesions; characterized by atypical CD4+ cells with “cerebriform” nuclei and intraepidermal
common in Japan (T-cell in Tokyo), West neoplastic cell aggregates (Pautrier microabscess). May progress to Sézary syndrome
Africa, and the Caribbean. (T-cell leukemia).
Lytic bone lesions, hypercalcemia.
Management Non-Hodgkin's lymphoma (NHL)
l follicular lymphoma
Ø stage IIA →local radiotherapy as first-line
Ø stage IIA + asymptomatic + single radiotherapy volume is not suitable→'watch and wait‘ (observation without therapy)
Ø stage IIA + symptomatic + single radiotherapy volume is not suitable→treat as advanced-stage (stages III and IV) symptomatic
Ø advanced-stage (stages III and IV) asymptomatic→rituximab
Ø advanced-stage (stages III and IV) symptomatic→rituximab + combination with:
u cyclophosphamide, vincristine and prednisolone (CVP)
u cyclophosphamide, doxorubicin, vincristine and prednisolone (CHOP)
u mitoxantrone, chlorambucil and prednisolone (MCP)
u cyclophosphamide, doxorubicin, etoposide, prednisolone and interferon‑ α (CHVPi) or
u chlorambucil
Ø Relapsed or refractory advanced-stage (stages III and IV) :
u induction of remission→ Rituximab + combination with chemotherapy
u maintenance therapy→ Rituximab monotherapy
u in second or subsequent remission→ stem cell transplantation
l MALT lymphoma
Ø H. pylori-positive gastric MALT lymphoma→ Helicobacter pylori eradication therapy
Ø H. pylori-negative gastric MALT lymphoma→ Helicobacter pylori eradication therapy
Ø gastric MALT lymphoma that responds clinically and endoscopically to H. pylori eradication therapy but who have residual disease shown
by surveillance biopsies of the stomach, + no high-risk features.→ 'watch and wait' (observation without therapy)
Ø residual MALT lymphoma after H. pylori eradication therapy + high risk of progression [H. pylori‑ negative at initial presentation or
t(11:18) translocation],
u chemotherapy (for example, chlorambucil or CVP) + rituximab OR
u gastric radiotherapy.
Ø Non-gastric MALT lymphoma
u localised disease sites→radiotherapy
u if radiotherapy is not suitable or disseminated disease→ chemotherapy (for example, chlorambucil or CVP) + rituximab
u localised + asymptomatic + radiotherapy is not suitable→ 'watch and wait‘ (observation without therapy)
l mantle cell lymphoma
Ø advanced-stage , symptomatic→chemotherapy + rituximab
Ø localised stage I or II → radiotherapy
Ø non-progressive + asymptomatic + radiotherapy is not suitable →'watch and wait‘ (observation without therapy)
Ø chemosensitive mantle cell lymphoma→ autologous stem cell transplantation
Ø previously untreated + stem cell transplantation is unsuitable →Bortezomib
Q50 55-year-old lady with 2 month history of a neck lymph
node, otherwise asymptomatic. Investigations showed Hb 12,
MCV 89, WBC 6.3, Platelet 355. Renal function normal. What
will you do next.
A. Biopsy of neck mass
B. Blood smear
C. PET-CT
D. Chest X ray
E. Platelet function

Q51: A 33 year old man presents with recent onset of


Q52
intermittent fever, night sweats and a left neck swelling.
Physical examination showed generalized painless rubbery
lymphadenopthy. Which of the following is the most
appropriate NEXT investigation?
A. Bone marrow examination
B. CT scan
C. PET scan
D. Fine needle aspiration of the lymph node
E. Excisional biopsy of the lymph node
Q53 Regarding malignant lymphomas:
A. 20% of those diagnosed in Hong Kong are Hodgkins type
B. 20% of those diagnosed in Hong Kong are T-cell type
C. Diffuse B-cell type are most common histology
D. 50 childhood cases in HK each year
E. 40% of patients are treated with bone marrow
transplantation

Q54 . an 18 yo man presents with fever, night sweats, weight


loss, physical exam shows rubber lymph node in neck. The best
management is:
A. Fine needle aspiration
B. Excisional biopsy of lymph node
C. Reassure
D. Recheck after 6 weeks
Q53
Fraction Key points Blood products
Packed red cells l Used for transfusion in chronic anaemia and cases where infusion of large volumes of fluid may result in cardiovascular compromise.
l Product obtained by centrifugation of whole blood.1U rise 1gHB/dL
l In a stable patient, red cell packs may be transfused over 90-120 minutes
l Rapid infusion of red cells or fresh frozen plasma may be required in an acutely bleeding patient but not in patient who is stable.
Platelet rich l Usually administered to patients who are thrombocytopaenic and are bleeding or require surgery.
plasma l It is obtained by low speed centrifugation.
Platelet l Prepared by high speed centrifugation
concentrate l administered to patients with thrombocytopaenia.
l the life span of transfused platelets is only 3-7 days.
l platelet transfusion should not take more than 20-30 minutes.
l Patients who are refractory to platelet transfusions:
Ø should be first investigated to check for adequate platelet rises. This is best done on a one or two-hour post platelet transfusion sample.
Ø Further test would include checking for HLA antibodies
Fresh frozen l Prepared from single units of blood.
plasma l Contains clotting factors, albumin and immunoglobulin.
l Unit is usually 200 to 250ml.
l Usually used in correcting clotting deficiencies in patients with hepatic synthetic failure who are due to undergo surgery.
l Usual dose is 12-15ml/Kg -1 .
l It should not be used as first line therapy for hypovolaemia.
Cryoprecipitate l Formed from supernatant of FFP.
l Rich source of Factor VIII and fibrinogen.
l Allows large concentration of factor VIII to be administered in small volume.
SAG-Mannitol Removal of all plasma from a blood unit and substitution with:
Blood • Sodium chloride
• Adenine
• Anhydrous glucose
• Mannitol
Up to 4 units of SAG M Blood may be administered. Thereafter whole blood is preferred. After 8 units, clotting factors and platelets should be
considered.
l plasma derivatives (such as factor VIII) are prepared from several thousand plasma donations, typically 20,000, or 5,000 kg of
plasma at a time.
l Pooled plasma has been sourced from outside the UK since 1999 to avoid vCJD risks.
Ø The process involves several chemical steps including:
u ethanol extraction,
u chromatography, and
u viral inactivation steps which results in a freeze-dried product.
l These products have a long shelf life of several months to years.
Cell saver devices
These collect patients own blood lost during surgery and then re-infuse it. There are two main types:
l Those which wash the blood cells prior to re-infusion. These are more expensive to purchase and more complicated to operate.
However, they reduce the risk of re-infusing contaminated blood back into the patient.
l Those which do not wash the blood prior to re-infusion.
Their main advantage is that they avoid the use of infusion of blood from donors into patients and this may reduce risk of blood
borne infection. It may be acceptable to Jehovah's witnesses. It is contraindicated in malignant disease for risk of facilitating
disease dissemination.
Blood products used in warfarin reversal
Immediate or urgent surgery in patients taking warfarin:
1. Stop warfarin
2. Vitamin K (reversal within 4-24 hours)
u IV takes 4-6h to work (at least 5mg)
u Oral can take 24 hours to be clinically effective
3. Fresh frozen plasma
u Used less commonly now as 1st line warfarin reversal
u 30ml/kg -1
u Need to give at least 1L fluid in 70kg person (therefore not appropriate in fluid overload)
u Need blood group
u Only use if human prothrombin complex is not available
4. Human Prothrombin Complex (reversal within 1 hour)
u Bereplex 50 u/kg
u Rapid action but factor 6 short half life, therefore give with vitamin K
Neonatal exchange transfusion
l An exchange transfusion requires blood which is plasma reduced whole blood in CPD (citrate phosphate
dextrose/anticoagulant), irradiated and less than five days old.
l The Rh group should either be Rh negative or identical to the neonate, to avoid haemolytic transfusion reaction in the neonate.
Blood product transfusion complications
Complications
l haemolytic: immediate or delayed
l febrile reactions
l transmission of viruses, bacteria, parasites, vCJD
l Hyperkalaemia
l iron overload
l ARDS
l clotting abnormalities
Immediate haemolytic reaction
l occur during the transfusion.
l e.g. ABO mismatch
l massive intravascular haemolysis
Delayed haemolytic transfusion reaction
l occurs 24 hours after the transfusion.
l This happens in a patient who has been previously immunised by transfusions or pregnancy. The antibodies are not detectable
initially but become obvious as a secondary immune response to the antigen exposure during the transfusion occurs.
Febrile reactions
l due to white blood cell HLA antibodies
l often the result of sensitization by previous pregnancies or transfusions
l Febrile non-haemolytic reactions are very common, and are due to the presence of pyrogenic cytokines released from leucocytes
during storage of the blood units.
Ø apart from a mild fever, the patient is very well.
u rapid rise in temperature may be due to ABO incompatibility, but With ABO incompatibility patients become shocked very
quickly.
Causes a degree of immunosuppression
l e.g. patients with colorectal cancer who have blood transfusions have a worse outcome than those who do not
The risk of viral transmission
l A broad knowledge of the risks may be required while consenting a patient for blood transfusion.
l in the United Kingdom, the risks;
Ø For hepatitis B are 1 per 1.3 million donations
Ø For HIV are 1 in 6.5 million and
Ø For hepatitis C 1 in 28 million donations.
Transmission of vCJD
l although the absolute risk is very small, vCJD may be transmitted via blood transfusion
l a number of steps have been taken to minimise this risk, including:
Ø from late 1999 onward, all donations have undergone removal of white cells (leucodepletion) in order to reduce any vCJD
infectivity present
Ø from 1999, plasma derivatives have been fractionated from imported plasma rather than being sourced from UK donors. Fresh
Frozen Plasma (FFP) used for children and certain groups of adults needing frequent transfusions is also imported
Ø from 2004 onward, recipients of blood components have been excluded from donating blood
iron overload
l secondary to chronic blood transfusion (eg : in myelodysplastic syndrome)
l early signs:
Ø grey skin
Ø early hear failure
Ø diabetes
l treatment:
Ø iron chelation with desferrioxamine subcutaneously
u bind iron
u needs to be given for 8 – 12 hours a day for 5 – 7 days per week
Ø common side effects of desferrioxamine:
u high frequency deafness
u Retinopathy
u Yersinia infection
irradiated blood products
l the advantage of irradiated red cells
Ø Inactivates donor lymphocytes
l Indications for irradiated blood products
Ø Those at risk of transfusion associated with graft versus host disease such as neonates
Ø Those receiving purine analogues based chemotherapy
Ø Hodgkin's lymphoma
Ø Immunodeficiency states
Ø Post bone marrow transplants
Pre-operative request for the blood bank for elective surgeries
l Group and save only
Ø A 'group and save' is adequate for elective surgeries and is standard practice in most modern blood banks. This will
involve blood grouping and its confirmation as well as an antibody screen.
Ø Other options include cross match and a direct Coombs' test are not routinely done for elective surgery
Transfusion errors
l Mislabelling of samples, requests, or wrongly identifying recipients are the commonest transfusion errors.
Definition
l (TRALI) is a rare but serious syndrome characterized by sudden acute respiratory distress within six hours after blood product
administration
Risk factors Transfusion Related Acute Lung Injury (TRALI)
l Caused by anti-HLA, Human Neutrophil Antigens (HNA) or anti-granulocytes antibody in donor blood.
l Donor’s blood sensitization occurs in:
Ø Multiparous ♀ develop these antibodies through exposure to fetal blood
Ø Previous transfusion
Ø Transplantation patient
l When blood is obtained from above mentioned donors, it carries higher risk for recipient to develop TRALI; those who have
lung pathology are more susceptible. TRALI symptoms resemble ARDS.
Pathophysiology
l transfused human leukocyte or neutrophil antigen (HLA or HNA) antibodies→ activation of recipient’s neutrophils→
Neutrophils adhere to pulmonary endothelium to increase permeability and cause pulmonary edema.
l Patients with certain clinical conditions (eg, infection, inflammation, surgery) have primed neutrophils that are susceptible to
activation by transfused bioactive substances.
l TRALI has two proposed pathophysiologic mechanisms:
1. the antibody hypothesis. (antigen-antibody interactions)
Ø The human leukocyte antigen (HLA class I, HLA class II) or human neutrophil antigen (HNA) antibody in the
transfused component reacts with neutrophil antigens in the recipient. The recipient's neutrophils lodge in the
pulmonary capillaries and release mediators that cause pulmonary capillary leakage.
Ø As a consequence, many patients with TRALI will develop transient leukopenia.
Ø However, transfusions of blood components containing neutrophil antibodies may cause leukopenia, that do not
meet the definition of TRALI.
2. The neutrophil priming hypothesis:
Ø does not require antigen-antibody interactions
Ø occurs in patients with clinical conditions that predispose to neutrophil priming and endothelial activation such as
infection, surgery, or inflammation.
Ø Bioactive substances in the transfused component activate the primed, sequestered neutrophils, and pulmonary
endothelial damage occurs.
l Both mechanisms lead to pulmonary edema in the absence of circulatory overload.
Feature
l Occurring within 1 to 6 hours of transfusion of plasma-containing blood components.
l Patients present with the rapid onset of dyspnea and tachypnea.
l There may be associated fever, cyanosis, and hypotension.
l Clinical examination reveals hypoxic respiratory distress, and pulmonary crackles may be present without signs of congestive
heart failure or volume overload.
l Chest x-ray (CXR) shows evidence of bilateral pulmonary edema unassociated with heart failure (non-cardiogenic pulmonary
edema), with bilateral patchy infiltrates, which may rapidly progress to complete "white out" indistinguishable from acute
respiratory distress syndrome (ARDS).
l Physiologic findings include acute hypoxemia with PaO2/FiO2 less than 300 mmHg and normal cardiac function on
echocardiogram.
Diagnosis:
 confirmed by finding of anti-HLA or anti-Neutrophil antibody in donors’ or recipient blood.
Treatment
l Early and intensive pulmonary support reduces the risk of a fatal outcome.
l Since the pulmonary edema in TRALI is not related to fluid overload or cardiac dysfunction, but to altered vascular
permeability in the lungs with exudation of fluid and protein into the alveoli, it is logical that:
Ø maintenance of adequate circulating volume is the most beneficial and appropriate therapy.
Ø Corticosteroids,
Ø Epinephrine
Ø and also ventilatory support are treatment options.
How to distinguish TRALI and ARDS from Pulmonary oedema?
l In the exam take into account the clinical findings and scenario to distinguish.
l The hallmark of ARDS is refractory hypoxia with non-cardiogenic pulmonary edema
l Normal pulmonary capillary wedge pressure is between 5 - 15 mmHg. A PCWP exceeding 15 mmHg suggests mitral stenosis,
mitral insufficiency, severe aortic stenosis, aortic regurgitation, ventricular failure, or other cardiac defects or pathologies.
l When the PCWP exceeds 20 mmHg, the transmission of this pressure back into the pulmonary vasculature increases
pulmonary capillary hydrostatic pressure which can lead to pulmonary oedema.
Q54 Patient requires transfusion, which of the following is done in the hospital blood
bank.
A. ABO typing of the donor’s red blood cell
B. Check for clinically significant antibodies in donor’s serum
C. Check for clinically significant antibodies in recipient’s serum
D. Mix the donor serum against recipient red blood cells
E. Mix the recipient serum against donor’s red blood cells

Q55 30/F with newly diagnosed aplastic anemia, received 6


packs of whole blood transfusion. Soon after completion of
transfusion, she developed shortness of breath, and oxygen
desaturation. What is the likely diagnosis?
A. AMI
B. pulmonary embolism
C. transfusion-related acute lung injury
D. Sepsis
E. acute hemolytic transfusion reaction
Q56 Which of the following products is most appropriate for hemophilia A?
A. whole blood
B. Fresh frozen plasma
C. Serum
D. Cryoprecipitate
E. Platelet rich plasma

Q57 Which of the following is not a indication for plasmapheresis?


A. Goodpasture’s syndrome
B. Guillain-barre syndrome
C. Myasthenia gravis
D. TTP
E. Aplastic anemia
Hematopoietic stem cell transplantation (HSCT) involves the intravenous (IV) infusion of autologous or allogeneic stem cells to reestablish hematopoietic function in
patients whose bone marrow or immune system is damaged or defective.
Common Indications for HSCT
Autologous Transplantation Allogeneic Transplantation

Malignant Disorders Nonmalignant disorders Malignant Disorders Nonmalignant disorders

Neuroblastoma Autoimmune disorders AML Aplastic anemia


Non-Hodgkin lymphoma Amyloidosis Non-Hodgkin lymphoma Fanconi anemia
Hodgkin lymphoma Hodgkin lymphoma Severe combined immunodeficiency
Acute myeloid leukemia Acute lymphoblastic leukemia Thalassemia major
Medulloblastoma Chronic myeloid leukemia Diamond-Blackfan anemia
Germ-cell tumors Myelodysplastic syndromes Sickle cell anemia
Multiple myeloma* Multiple myeloma* Wiskott-Aldrich Syndrome
*Uncommon in children; common reasons for transplantation in Chronic lymphocytic leukemia* Osteopetrosis
adults CLinicaL Staging and grading of acute graft-Versus-host disease Inborn errors of metabolism
Complications following Autoimmune disorders
hematopoietic cell transplant
Graft-Versus-Host Disease GVHD is the
Early Direct Chemoradiotoxicities ;
result of allogeneic T cells that are
Late Direct Chemoradiotoxicities ;
transferred with the donor’s stem cell
Graft Failure ;Graft-Versus-Host
inoculum reacting with antigenic targets
Disease ;Infection
on host cells. Acute GVHD usually occurs
within the first 3 months after transplant
with a peak onset around 4 weeks and is
characterized by an erythematous
The incidence of acute GVHD is higher in recipients of stem cells maculopapular rash; by persistent
from mismatched or unrelated donors, in older patients, and in anorexia or diarrhea, or both; and by liver
patients unable to receive full doses of drugs used to prevent the
disease. One general approach to the prevention of GVHD is the disease with increased serum levels of
administration of immunosuppressive drugs early after transplant. bilirubin, alanine and aspartate
Combinations of methotrexate and either cyclosporine or
tacrolimus are among the most effective and widely used regimens. aminotransferase, and alkaline
Prednisone, anti–T cell antibodies, mycophenolate mofetil, sirolimus, phosphatase. Because many conditions
and other immunosuppressive agents have also been or are being can mimic acute GVHD, the diagnosis
studied in various combinations. Despite prophylaxis, significant
acute GVHD will develop in ~30% of recipients of stem cells from usually requires skin, liver, or endoscopic
matched siblings and in as many as 60% of those receiving stem biopsy for confirmation. In all these
cells from unrelated donors. The disease is usually treated with
glucocorticoids, additional immunosuppressants, organs, endothelial damage and
or monoclonal antibodies targeted against T cells or T cell subsets. lymphocytic infiltrates are seen
Cetuximab
Flavopiridol

Ipilimumab

imetelstat sodium

尼拉帕利是一种口服、每日
一次的聚ADP核糖聚合酶
(PARP)抑制剂
TGF/IGB-F inhibitors

Classes of cytotoxic drugs


Bevacizumab
l Alkylating agents: Anti-proliferative drugs that bind via alkyl groups to DNA leading to apoptotic cell death, eg cyclophosphamide, chlorambucil, busulfan.
l Angiogenesis inhibitors: Eg bevacizumab, aflibercept, sunitinib.
l Antimetabolites: Interfere with cell metabolism including DNA and protein synthesis, eg methotrexate, 5 -fl uorouracil.
l Antioestrogens: Aromatoase inhibitors (eg letrozole, anastrozole), oestrogen receptor antagonists (eg tamoxifen, raloxifene) used in breast cancer treatment.
l Antitumour antibiotics: Interrupt DNA function, eg dactinomycin, doxorubicin, mitomycin, bleomycin.
l Monoclonal antibodies: Antibodies to a specific tumour antigen can slow tumour growth by enhancing host immunity, or be conjugated with chemotherapy/radioactive isotopes to allow
targeted treatment. Expect more of these in future.
l Topoisomerase inhibitors: Interrupt regulation of DNA winding, eg etoposide.
l Vinca alkaloids and taxanes: ‘Spindle poisons’ which target mechanisms of cell division, eg vincristine, vinblastine, docetaxel.
Side-eff ects
Due to cytotoxic effects on non-cancer cells. Greatest effect seen on dividing cells, ie gut, hair, bone marrow, gametes
l Vomiting: Prophylaxis given with most cytotoxic regimens
l Alopecia: May profoundly impact quality of life. Consider ‘cold-cap’, wig services.
l Neutropenia: Most commonly seen 7–14 d after chemotherapy. Neutropenic sepsis is life-threatening and needs urgent assessment and empirical treatment
紫杉醇促进微管蛋白二聚体的聚合并阻止其解聚而达到稳定微管的作用
长春新碱主要抑制微管蛋白的聚合而影响纺锤体微管的形成
赫赛汀
Q60 Which of the following has cardiotoxicity?
A. Doxorubicin
B. Vincristine
C. Cisplatin
D. Cyclophosphamide
Q58 E. Bleomycin

Q59
Azathioprine and 6-MP are
metabolized by xanthine oxidase;
thus both have risk of toxicity
with allopurinol or febuxostat.
Diagnostically Guided Protein Kinase Antagonists Multikinase inhibitors
Imatinib Bcr-Abl fusion protein (CML/ALL); c-kit mutants, PDGFR variants (GI stromal Sorafenib Renal cell, hepatocellular, differentiated thyroid carcinoma
tumor; eosinophilic syndromes)
Nilotinib Bcr-Abl fusion protein (CML) and some imatinib-resistant variants Pazopanib Renal cell carcinoma, soft tissue sarcoma

Dasatinib Bcr-Abl fusion protein (CML/ALL); wild-type and imatinib-resistant mutants Regorafenib Second-line colorectal cancer; GI stromal tumor

Bosutinib Bcr-Abl fusion protein (CML); wild-type and imatinib-resistant mutants Sunitinib Renal cell carcinoma, pancreatic neuroendocrinetumor, GI stromal
Ponatinib T315I mutation of Bcr-Abl fusion protein (CML) Vandetanib Medullary thyroid cancer
Gefitinib First-line treatment of NSCLC with ATP site mutation of EGFR Cabozantinib Medullary thyroid cancer

Erlotinib First-line treatment of NSCLC with ATP site mutation of EGFR; second-line Axitinib Renal cell carcinoma, second line
treatment of wild-type EGFR NSCLC
Afatinib First-line treatment of NSCLC with ATP site mutation of EGFR Proteasome Inhibitors

Crizotinib EML4-Alk fusion protein Bortezomib Multiple myeloma, mantle cell lymphoma

Lenvatinib Hepatocellular carcinoma Carfilzomib Multiple myeloma, second line

Vemurafenib BRAF V600E in melanoma Histone Deacetylase Inhibitors

Dabrafenib BRAF V600E in melanoma Vorinostat Cutaneous T cell lymphoma, second line

Trametinib BRAF V600E in melanoma (both as single agent and in combination with Romidepsin Cutaneous T cell lymphoma, second line
dabrafenib)
Diagnostically Guided Retinoid mTOR Inhibitors

Tretinoin APL t(15,17) Temsirolimus Renal cell carcinoma, second line or poor prognosis

Arsenic trioxide APL t(15,17) Everolimus Renal cell carcinoma, advanced; subependymal giant-cell
astrocytoma; breast cancer, hormone receptor positive, resistant to
antiestrogen; pancreatic neuroendocrine
Antibodies used In Cancer treatment

利妥昔单抗/美罗华

奥法木单抗

曲妥珠单抗/赫赛汀

帕妥珠单抗/帕捷特

西妥昔单抗/爱必妥

帕尼单抗

贝伐珠单抗/安维汀

阿仑单抗

伊匹单抗

帕博利珠单抗/可瑞达
Monoclonal antibody to programmed cell death-1 protein (PD-1)
Keytruda Melanoma, Lung Cancer, Head & Neck Squamous Cell Carcinoma, Hodgkin Lymphoma
Tumors possess a microenvironment (tumor stroma) with immune cells including pembrolizumab 帕博利珠单抗/可瑞达
both helper T cells, suppressor T cells (both “regulatory” of other immune cell Opdivo Melanoma, Lung Cancer, Head & Neck Squamous Cell Carcinoma, Hodgkin Lymphoma Renal Cell
function), macrophages, and cytotoxic T cells. Cytokines found in the stroma and Nivolumab Carcinoma; Hepatocellular Carcinoma 纳武利尤单抗/:欧狄沃
deriving from macrophawsges and regulatory T cells modulate the activities of
Libtayo Cutaneous Squamous Cell Carcinoma
cytotoxic T cells, which have the potential to kill tumor cells. Antigens released by cemiplimab
tumor cells are taken up by Antigen Presenting Cells (APCs), also in the stroma.
Antigens are processed by the APCs to peptides presented by the Major Monoclonal antibody to programmed cell death ligand-1 protein (PDL1)
Histocompatibility Complex to T-cell antigen receptors, thus providing an (+) Tecentriq Urothelial Carcinoma; Lung Cancer; Triple-Negative Breast Cancer
activation signal for the cytotoxic tumor cells to kill tumor cells bearing that antigen. atezolizumab
Negative (–) signals inhibiting cytoxic T cell action include the CTLA4 receptor (on T
Imfinizi Urothelial Carcinoma; Lung Cancer; Triple-Negative Breast Cancer
cells), interacting with the B7 family of negative regulatory signals from APCs, and
durvalumab
the PD receptor (on T cells), interacting with the PD-L1 (–) signal coming from tumor
cells expressing the PD-1 ligand (PD-l1). As both CTLA4 and PD1 signals attenuate the Bavencio Merkel Cell Carcinoma; Urothelial Carcinoma; Renal Cell Carcinoma
anti-tumor T cell response, strategies which inhibit CTLA4 and PD1 function are a avelumab
means of stimulating cytotoxic T cell activity to kill tumor cells. Cytokines from other Recombinant, human cytotoxic T-lymphocyte antigen 4 (CTLA-4) - blocking antibody
immune cells and macrophages can provide both (+) and (–) signals for T cell action,
Ipilimumab Malignant Melanoma; Colorectal Cancer; renal cell carcinoma
and are under investigation as novel immunoregulatory therapeutics.
The immunomodulatory drugs (IMiD) (thalidomide, lenalidomide, pomalidomide) appear to have multiple actions, including modulation of T
cells and NK cells, adhesion, anti-angiogenesis, and direct anti-tumor activity. The agent has shown promising activity in several hematologic
malignancies (MDS, MM, CLL, NHL) and is FDA approved for MDS (5q-) and MM. Combination strategies are being evaluated in MDS, AML, and
MM that would further broaden the therapeutic potential of lenalidomide as a bone marrow microenvironment modulating agent. Lenalidomide
is also being evaluated in refractory solid tumors (prostate cancer, NSCLC, melanoma, RCC, ovarian carcinoma).
A humanized Mab to CD200 (Alexion), an innate immune modulator, is currently in phase I studies for CLL and MM. Many vaccine trials have
been conducted against targets in NHL, melanoma, prostate cancer, PDA, and ovarian cancer without a valid signal to pursue such strategies as
anti-cancer agents. Despite these disappointments, targeting immune dysregulation in cancer is an active area of research, and future
investigations should provide the necessary tools to fight cancer.
Bavituximab (Peregrine Pharmaceuticals) is a Mab that binds to phosphatidylserine (PS磷脂酰丝氨酸), which is normally located in the inner
membrane of cells but becomes exposed to the outside of the cells that line the blood vessels of tumors, creating a specific target for anti-cancer
therapy. Binding of bavituximab to PS helps mobilize the body's immune system to destroy the tumor and associated blood vessels.
Paraneoplastic syndromes consist of symptoms attributable to a malignancy mediated by hormones, cytokines, or the cross-
reaction of tumour antibodies. They do not correlate with stage/prognosis and may pre-date other cancer symptoms.
Paraneoplastic syndrome Comment Malignancies

Hypercalcaemia Parathyroid hormone-related protein secreted by tumour Lung, oesophagus, skin, cervix, breast, kidney
SIADH ( ADH ) secretion causing ↓Na+ Lung, pancreas, lymphomas, prostate
Cushing’s syndrome secretes ACTH or CRF , causing adrenal to produce high levels of Lung, pancreas, thymus, carcinoid
corticosteroid
Neuropathy Antibody-mediated neuronal degeneration: peripheral, autonomic, Lung, breast, myeloma, Hodgkin’s, GI
cerebellar
Lambert–Eaton myasthenic Antibody to voltage-gated ion channel on pre-synaptic membrane causes Mostly lung. Also GI , breast, thymus
syndrome weakness (proximal leg most common)
Dermatomyositis & polymyositis Inflammation of the muscles + /- heliotrope rash Lung, breast, ovary, GI

Acanthosis nigricans Velvety, hyperpigmented skin (usually flexural) GI

Pemphigus Blisters to skin/mucous membranes Lymphoma, thymus, Kaposi’s sarcoma


Hypertrophic osteoarthropathy Periosteal bone formation, arthritis, and finger clubbing Lung
Sweet syndrome : acute myeloid leukemia
Sweet syndrome is characterized by fever, neutrophilia, and sterile
erythematous plaques or nodules that respond to steroid therapy. the
upper extremities and face the syndrome may well be driven by T-cell
abnormalities. abnormalities in neutrophil chemotaxis, autoantibodies
directed against neutrophils, and alteration in cytokine levels . skin lesions
in the syndrome do not directly involve malignant cells. Systemic steroid
therapy and/or treatment of the underlying malignancy

Acquired ichthyosis (AI) manifest with small, whitish to


brownish, polygonal scales that lift up at the free edge and are
widely distributed on the trunk and extensor surfaces of the
extremities. The palms and soles are usually spared. This
condition is associated with a variety of chronic illnesses,
including malignancy. AI is most strongly associated with
Hodgkin lymphoma. an autoimmune response directed against
the skin and the secretion of keratinocyte growth factors by
solid tumors. Paraneoplastic AI responds to successful
treatment of the underlying malignancy. The mainstay of
symptomatic treatment of AI includes hydration of the skin
and prevention of evaporation. This is achieved through the
topical application of alpha-hydroxy acids (eg, lactic, glycolic,
and pyruvic acids) and emollients (eg, urea creams, propylene
glycol). Topical corticosteroids are not useful.
erythema gyratum repens (EGR), which is derived from the Latin repens, meaning creep, gyrate (circular) Of all gyrate (circular)
erythemas, EGR is almost always (>80%) indicative of an underlying malignancy. Lesions consist of dramatic, erythematous,
concentric rings with scales, which cause the classic wood-grain appearance. Lesions may be flat or slightly raised. They are
Acanthosis nigricans (AN) manifests as a localized to the trunk and proximal extremities, sparing the feet, hands, and facethe lesions may advance at a rate of 1cm daily.
hyperpigmented, velvety thickening of the skin that Patients universally report severe pruritus, and concurrent hyperkeratosis of the palms and soles has also been reported.
usually occurs in the intertriginous zones, including the Marked peripheral eosinophilia may occur. Thus, EGR is a clinically specific paraneoplastic syndrome. EGR occurs more
axillae, groin, neck, and inframammary folds. frequently in males, whites, and elderly persons. Histopathologic features are nonspecific and include perivascular
histopathologic changes of hyperkeratosis and lymphocytic infiltrates with focal spongiosis and parakeratosis. Among patients with EGR without a detectable malignancy,
diagnoses of tuberculosis, CREST syndrome, and pregnancy are thought to be related.
papillomatosis of the epidermis. Acanthosis is seldom
present, and hyperpigmentation is related to
hyperkeratosis, not melanin deposition. AN occurring in
hyperinsulinemic states and a spectrum of autoimmune
disease. Most cases of paraneoplastic AN involve an
adenocarcinoma, most commonly one arising in the
gastrointestinal tract (stomach or live. Activation of
signaling pathways((TGF)–alpha) by binding of growth
factors to specific skin receptors could play a primary
role in the pathogenesis of paraneoplastic AN Necrolytic migratory erythema The skin and mucous membranes are affected, with painful, migratory, erythematous, polycyclic
Treatment should be directed at the associated patches or plaques with superficial pustules, vesicles, or bullae being present. The lesions are typically widespread, but they are
accentuated in the intertriginous areas, lower abdomen, and proximal parts of the lower extremities. It is often present in a triad
underlying condition
with NME, glucose intolerance, and hyperglucagonemia. It is commonly complicated by Candida albicans or Staphylococcus
aureus infection. Confirming a diagnosis of paraneoplastic NME is achieved by performing specific laboratory tests and imaging
studies to localize the pancreatic tumor. Elevated serum glucagon levels (and occasionally insulin levels) Surgical resection or
effective chemotherapy of the tumor is accompanied by resolution of the dermatitis.
Paraneoplastic syndromes affecting nervous system
Lambert-Eaton myasthenic syndrome
l associated with small cell lung cancer (also breast and ovarian)
l antibody directed against pre-synaptic voltage gated calcium channel in the peripheral nervous system
l can also occur independently as autoimmune disorder
Anti-Hu
l associated with small cell lung carcinoma and neuroblastomas
l sensory neuropathy - may be painful
l cerebellar syndrome
l encephalomyelitis
Anti-Yo
l associated with ovarian and breast cancer
l cerebellar syndrome
Anti-GAD antibody
l associated with breast, colorectal and small cell lung carcinoma
l stiff person's syndrome or diffuse hypertonia
Anti-Ri
l associated with breast and small cell lung carcinoma
l ocular opsoclonus-myoclonus
Anti-Purkinje cell antibodies
l subacute cerebellar degeneration
l peripheral neuropathy due to a remote (autoimmune) effect of gynecologic or breast carcinoma
(GM1 antibodies) (Glycolipid ganglioside-monosialic acid) associated with
l Lower motor neuron syndromes
l Amyotrophic lateral sclerosis
l Multiple sclerosis
l Other multifocal neuropathies and
l Systemic lupus erythematosus (SLE) with central nervous system involvement.
Q64 Ovarian cancer + unsteadiness, nystagmus and past pointing.
Which antibody is most likely to be present?
A. Anti-Hu
B. Anti-Yo
C. Anti-Ri
D. GM1 antibodies
E. Anti-GAD antibody

Q65 68 year-old man, heavy smoking for 40 years. Intermittent dry cough, weight
loss, difficult to rise from chair and climb stairs, low extremities muscle pain for 3
months. On examination, left cervical painless lymph nodes. Low extremity
proximal weakness, grade 4—,strength improves after exercise. Lab:Hb 16,
plt112, CK 106 ESR 15. regarding the underlying diagnosis, which of the following
will be positive?
A. Anti-voltage-gated Ca2+ channel
B. Anti- AChR antibodies
C. MuSK antibodies
D. anti- Mi2
E. anti- Jo1
Tumour lysis syndrome (TLS) is a potentially deadly condition
causes:
l treatment of high grade lymphomas and leukaemias.
l It can occur in the absence of chemotherapy but is usually triggered by the introduction of combination chemotherapy.
Pathophysiology:
l breakdown of the tumour cells and the subsequent release of chemicals from the cell.
Features:
l high potassium
l high phosphate
l low calcium.
l It should be suspected in any patient presenting with an acute kidney injury in the presence of a high phosphate and high uric acid level.
Diagnosis:
l From 2004 TLS has been graded using the Cairo-Bishop scoring system -Laboratory tumor lysis syndrome: abnormality in two or more of the
following, occurring within three days before or seven days after chemotherapy.
Ø uric acid > 475umol/l or 25% increase
Ø potassium > 6 mmol/l or 25% increase
Ø phosphate > 1.125mmol/l or 25% increase
Ø calcium < 1.75mmol/l or 25% decrease
l Clinical tumor lysis syndrome: laboratory tumor lysis syndrome plus one or more of the following:
Ø increased serum creatinine (1.5 times upper limit of normal)
Ø cardiac arrhythmia or sudden death
Ø seizure
Prevention:
l Patients at high risk of TLS should be given IV allopurinol or IV rasburicase immediately prior to and during the first days of chemotherapy.
Ø Rasburicase is a recombinant version of urate oxidase, an enzyme that metabolizes uric acid to allantoin. Allantoin is much more water
soluble than uric acid and is therefore more easily excreted by the kidneys.
u The commonest reported side effect of rasburicase is fever.
u rasburicase overdose may lead to accumulation of hydrogen peroxide.
l patients at low risk → oral allopurinol during chemotherapy
l Other options for the management of tumour lysis syndrome include
Ø Acetazolamide to drive urine alkalinisation.
Q66 25 year old male with a history of disseminated
nonseminomatous testicular cancer presented with malaise.
Below is his blood results:
K 6.8 (did not give ref range), High urea, High creatinine, low
corrected calcium, high phosphate, LDH 2000
What is the cause of his condition?
A. Lab error
B. Poisoning
C. Tumour lysis syndrome
D. Paraneoplastic
E. Acute renal failure

Q67 About tumor lysis syndrome, which of the following is wrong?


A. High uric acid
B. High potassium
C. High calcium
D. High phosphate
E. High creatinine
Palliative care prescribing: pain
NICE guidelines (2012) and SIGN guidelines (2008)
l metastatic bone pain may respond to NSAIDs, bisphosphonates or radiotherapy
Starting morphine
l Morphine is the opioid of choice for treating moderate to severe cancer pain.
l Choices between morphine preparations
Ø when starting treatment, offer patients with advanced and progressive disease regular oral modified-release (MR) or oral
immediate-release morphine(IR) (depending on patient preference), with oral immediate-release morphine for
breakthrough pain
Ø oral modified-release morphine should be used in preference to transdermal patches
Ø Immediate release preparations are used for titration as they offer greatest flexibility. Most patients should be started on
5-10mg orally every 4-hours, with the same dose prescribed as a breakthrough (or ‘rescue’) dose wherever needed. Once
drug requirements are constant, the patient can be converted to modified-release morphine.
Ø Once a patient has been titrated on immediate release opioids these can be converted to the equivalent dose of a
modified release preparation.
Ø If a patient has good pain control on one drug, the modified release version of this drug should be used.
l Morphine doses
Ø if no comorbidities use 20-30mg of MR a day with 5mg morphine for breakthrough pain. For example, 15mg modified-
release morphine tablets twice a day with 5mg of oral morphine solution as required
Ø When increasing the dose of opioids the next dose should be increased by 30-50%.
Ø An appropriate starting dose of morphine sulphate immediate release (IR) should not be more than 10mg every 4 hours.
Alternatively, morphine sulphate modified release (MR) 30mg 12 hourly could be used.
Opioids Side effects:
l Constipation: laxatives should be prescribed for all patients initiating strong opioids
Ø Morphine causes constipation by enhancing intestinal ring contractions. This results in hypersegmentation which in turn
impairs peristalsis.
Ø 90% of patients taking morphine require a laxative and a stimulant is the best choice (such as senna). Senna is the most
commonly used laxative for this indication
l Nausea: patients should be advised that nausea is often transient. If it persists then an antiemetic should be offered
l drowsiness is usually transient - if it does not settle then adjustment of the dose should be considered
Preferred opioids for patients with chronic kidney disease
l Opioids should be used with caution in patients with chronic kidney disease. Alfentanil, buprenorphine and fentanyl are
preferred
Ø Fentanyl patches are difficult to titrate because they are used for 72 hours. therefore only used once a patient has a
stable opiate usage.
Ø Fentanyl is a selective µ receptor agonist.
Ø It has extensive first-pass metabolism so is not especially effective orally.
Ø However, buccal absorption is good so lozenges are an effective mode of administration and have a rapid onset of action
(five minutes). This is therefore very useful for patients with "breakthrough pain".
Ø It is very useful in renal failure as it is metabolised mainly in the liver and it has inactive metabolites.
Combination therapies antagonism
l Partial opioid agonists (for example, buprenorphine), when used in association with morphine, may produce a reduction in the
analgesic effect due to partial antagonism.
l This is an aspect of pain management that needs to be considered when using combination therapies.
Oxycodone
l Oxycodone is often used as a second line opioid for patients who experience either inadequate analgesia or excessive side
effects with morphine.
l It has similar analgesic properties to morphine but is twice as potent.
l It is available in immediate-release and modified- release oral preparations and can also be used parentally.
l Parental oxycodone is twice as potent as oral oxycodone.
l The total daily dose of immediate and modified release oral oxycodone is the same.
l Oxycodone generally causes less sedation, vomiting and pruritis than morphine but more Constipation
Transdermal perparations:
The current BNF gives the following conversion factors for transdermal perparations
l a transdermal fentanyl 12 microgram patch equates to approximately 30 mg oral morphine daily
l a transdermal buprenorphine 10 microgram patch equates to approximately 24 mg oral morphine daily.
Diamorphine
Concerning diamorphine elixir for the relief of pain in terminal patients:
l Diamorphine has a rapid onset so could be used for breakthrough pain if the renal function is normal.
l Constipation is a characteristic sequel to treatment
l Hallucinations also tend to occur.
l An aperient (laxative) should always be added to the treatment regime.
l Addiction is not a problem.
l An intramuscular injection is three times more effective than the same oral dose.
l the best option for controlling pain associated with vomiting in palliative care → Subcutaneous diamorphine by continuous
infusion(able to effectively titrate the dose to achieve adequate analgesia)
l Nifedipine helps relieve painful oesophageal spasm and tenesmus associated with gastrointestinal tumours and could be used to relieve
odynophagia.
l Bisphosphonates are useful adjuncts for bone pain, especially in breast cancer and myeloma.
Ø Clodronate inhibits osteoclastic bone resorption and is used to treat malignant bone pain and the associated hypercalcaemia.
Ø The risk of osteonecrosis of the jaw is much greater for patients receiving intravenous bisphosphonates in the treatment of cancer.
u All patients receiving bisphosphonates for cancer should have a dental check-up before bisphosphonate treatment.
u other patients who are prescribed bisphosphonates should have a dental examination only if they have poor dental health.
Ø The beneficial effect of bisphosphonates can be delayed for up to two weeks and can last for one month, and treatments are therefore
usually given monthly (typically for 6 months).
Ø increase analgesia while waiting for the bisphosphonates to work and review over the next few days to see whether you could reduce
them again.
l Corticosteroids are used to treat pain from central nervous system tumours
l Oxybutynin: painful bladder spasm may be relieved by oxybutynin.
l Hyoscine:
Ø to reduce air way secretions in palliative care→ Both hyoscine and atropine when given subcutaneously are thought to be equally
appropriate for drying up secretions.
Ø hyoscine s/c can be given up to three times per day in boluses of 10-20 mg.
l Haloperidol
Ø Haloperidol is a first line antiemetic for opioid-induced nausea in the palliative care setting.
Ø Its action is predominantly via D2-receptor antagonism in the chemoreceptor trigger zone (CTZ) in the brain stem.
Ø 90% of patients taking morphine require antiemetics (morphine stimulates D2 receptors in the CTZ). Cyclizine, although commonly used,
is less effective.
Ø haloperidol→ dopamine receptor antagonist (D2) activity→ drug-induced parkinsonism (DIP).
l Cyclizine
Ø Cyclizine is a commonly used antihistamine antiemetic and its primary site of action is the vomiting centre (which is rich in histamine and
muscarinic cholinergic receptors).
Ø Cyclizine has a strong affinity for muscarinic receptors and therefore anticholinergic side effects (dry mouth, drowsiness, blurred vision,
constipation, etc) are common, especially in the first few days.
l Ondansetron
Ø Ondansetron is a 5HT 3 antagonist and is mainly used in post-chemotherapy or radiotherapy induced nausea.
Ø In the United Kingdom 5HT 3 antagonists are licensed only for post-chemotherapy and post-operative nausea.
Ø Which antiemetics is most useful following treatment with a platinum based chemotherapy? → Ondansetron
u Examples of platinum based chemotherapies are cisplatin, carboplatin and oxaliplatin
l Opioid toxicity
Ø Reduced conscious level, hallucinations, vomiting, myoclonic jerks and pinpoint pupils are features of opioid toxicity.
l Pain management
Ø The pain may be due to metastatic deposits within the ribs, but may also have an element of neuropathic pain. Nerve
pain often also has a nociceptive opioid responsive element and hence opioids (with a combination of nonsteroidal anti-
inflammatory drugs [NSAIDs]) should be tried first (eg: ibuprofen and tramadol) and used as part of the WHO analgesic
ladder. Morphine would be tried next, followed by the other agents.
l Gabapentin
Ø Gabapentin is a commonly used adjunctive agent for neuropathic pain.
Ø mechanism of action : (Activation of GABA inhibitory system).
Ø Four to six weeks of treatment are often needed before the patient experiences benefit.
l Hypercalcaemia
Ø Hypercalcaemia is a common problem in palliative care.
Ø prostate cancer with bone metastasis is a frequent cause.
Ø Hypercalcaemia can cause constipation (a constipation in cancer→ do blood tests, including bone profile )
l Codeine
Ø The analgesic effect of codeine depends on its conversion to morphine by the CYP2D6 hepatic enzyme. Up to 10% of
Caucasians are CYP2D6 poor metabolisers and are unlikely to derive any analgesia from it.
Ø If hepatic metabolise is impaired for any other reason (drugs or hepatic impairment) patients are also unlikely to benefit
from codeine.
Acupuncture is playing an increasing role in pain management. Which structures are involved in mediating the effects of
acupuncture? Cerebral cortex and A beta nerve fibres.
l The A beta nerve fibres are the path for fast transmission of sensation.
l Acupuncture also has a central effect.
Palliative care prescribing: agitation and confusion
l Underlying causes of confusion need to be looked for and treated as appropriate, for example hypercalcaemia, infection,
urinary retention and medication.
l If specific treatments fail then the following may be tried:
Ø first choice: haloperidol
Ø other options: chlorpromazine, levomepromazine
Ø In the terminal phase of the illness then agitation or restlessness is best treated with midazolam
Palliative care prescribing: hiccups
l Management of hiccups
Ø chlorpromazine is licensed for the treatment of intractable hiccups
Ø haloperidol, gabapentin are also used
Ø dexamethasone is also used, particularly if there are hepatic lesions
Palliative care: end of life care
l Glucocorticoids are prominent in end of life care not only for control of pain, but also to relieve nausea and fatigue and
improve general feelings of wellbeing. Randomized controlled trials have suggested only approximately a 15% improvement in
pain, although other benefits including reduced nausea and improve energy are seen. Dexamethasone is the usual agent of
choice.
Q68 What is the most appropriate opioid to prescribe for a syringe driver
in end stage cancer patient with renal failure?
A. Alfentanil
B. Buprenorphine
C. Fentanyl
D. Morphine
E. Diamorphine

Q69 metastatic bone pain may respond to?


A. NSAIDs
B. Bisphosphonates
C. Radiotherapy
D. Morphine
E. All of above

Q70 Most common used laxative for morphine-induced constipation?


A. Lactulose
B. Senna
C. Macrogol
D. Bran powder
E. Liquid paraffin
Q71 Which antiemetics is most useful following treatment with cisplatin?
A. Metoclopramide
B. Haloperidol
C. Ondansetron
D. Cyclizine
E. chlorpromazine

Q72 Which of the following is the feature of opioid toxicity?


A. Unconscious
B. Hallucinations
C. Myoclonic jerks
D. pinpoint pupils
E. All of the above

Q73 Which of the following is the prominent options for


palliative care for the end of life?
A. Morphine
B. Diamorphine
C. Midazolam
D. Dexamethasone
E. haloperidol
http://lmchk.hongjingedu.com/

You might also like